Você está na página 1de 55

Módulo de Combinatória

F
1
2

2
X Y

c Bruno Holanda. Este trabalho representa um conjunto de notas de aulas e deve ser usado apenas para
uso pessoal. Reproduzir qualquer parte deste material sem o consentimento do autor não é permitido.
brunolholanda@yahoo.com.br
Sumário

1 Idéias Introdutórias 5
1.1 Afinal, O que é um Problema de Olimpı́ada? . . . . . . . . . . . . . . . . . . . . . . . . . . . 5
1.2 Organizando as Informações . . . . . . . . . . . . . . . . . . . . . . . . . . . . . . . . . . . . . 6
1.3 Tipos de Soluções . . . . . . . . . . . . . . . . . . . . . . . . . . . . . . . . . . . . . . . . . . . 8

2 Contagem Simples e Permutações 11


2.1 Contagem Direta . . . . . . . . . . . . . . . . . . . . . . . . . . . . . . . . . . . . . . . . . . . 11
2.2 Separando em Casos . . . . . . . . . . . . . . . . . . . . . . . . . . . . . . . . . . . . . . . . . 15
2.3 Contagens Múltiplas . . . . . . . . . . . . . . . . . . . . . . . . . . . . . . . . . . . . . . . . . 17
2.4 Números Binomiais . . . . . . . . . . . . . . . . . . . . . . . . . . . . . . . . . . . . . . . . . . 18

3 Jogos 21
3.1 Simetria . . . . . . . . . . . . . . . . . . . . . . . . . . . . . . . . . . . . . . . . . . . . . . . . 21
3.2 Posições Vencedoras . . . . . . . . . . . . . . . . . . . . . . . . . . . . . . . . . . . . . . . . . 23
3.3 Idéias Avançadas . . . . . . . . . . . . . . . . . . . . . . . . . . . . . . . . . . . . . . . . . . . 25

4 Casa dos Pombos 26

5 Grafos 31
5.1 Conseitos Básicos . . . . . . . . . . . . . . . . . . . . . . . . . . . . . . . . . . . . . . . . . . . 32
5.2 Grafos Simples . . . . . . . . . . . . . . . . . . . . . . . . . . . . . . . . . . . . . . . . . . . . 34
5.3 Árvores . . . . . . . . . . . . . . . . . . . . . . . . . . . . . . . . . . . . . . . . . . . . . . . . 35

6 Tabuleiros 40
6.1 Introdução . . . . . . . . . . . . . . . . . . . . . . . . . . . . . . . . . . . . . . . . . . . . . . . 40
6.2 Os Poliminós . . . . . . . . . . . . . . . . . . . . . . . . . . . . . . . . . . . . . . . . . . . . . 40
6.3 Problemas Propostos . . . . . . . . . . . . . . . . . . . . . . . . . . . . . . . . . . . . . . . . . 42

7 Invariantes 46
7.1 Analisando as invariantes . . . . . . . . . . . . . . . . . . . . . . . . . . . . . . . . . . . . . . 46
7.2 Restos como invariantes . . . . . . . . . . . . . . . . . . . . . . . . . . . . . . . . . . . . . . . 48
7.3 Semi-invariantes . . . . . . . . . . . . . . . . . . . . . . . . . . . . . . . . . . . . . . . . . . . 49
SUMÁRIO 3

8 Princı́pio do Extremo 53
Prefácio

Estudar matemática não é fácil. Principalmente quando estamos estudando para as provas das olimpı́adas,
como a OBM, a Rio-platense, a Olimpı́ada de Maio e muitas outras que fazem parte da vida dos olı́mpicos
iniciantes. Além disso, a cada ano que se passa as provas ficam mais e mais difı́ceis para os alunos, e tudo
piora se o olı́mpico não possuir base suficiente para resolver os problemas. Foi pensando nisso que esta
apostila foi criada.

Sobre Este Material

Ao todo, esta apostila está dividida em oito capı́tulos e possui um total de 233 problemas propostos.
Como é um documento ainda inacabado, está repleto de erros. Tais erros podem (e devem) ser comunicados
por e-mail através do seguinte endereço: brunolholanda@yahoo.com.br Envie também problemas e sug-
estões.

Aos Alunos

Alguns dos problemas são bastante difı́ceis, mas não desista! Encare-os como desafios e como um treino
para as provas futuras. Se mesmo assim você não conseguir resolvê-los procure o professor de olimpı́ada mais
. .
próximo de você. Temos certeza que existe um na esquina de seu bairro. ⌣

Bruno Holanda,
Fortaleza CE
Janeiro 2007.
-1-
Idéias Introdutórias

Neste primeiro capı́tulo você terá uma idéia geral sobre o que vamos encontrar no restante deste trabalho.
Por este motivo, os problemas que seguem apresentam diferentes nı́veis de dificuldade. Não se preocupe se
você tiver que se esforçar mais para entender alguns exemplos e resolver alguns exercı́cios. Nosso objetivo é
nos aproximar dos problemas de olimpı́ada e aprender as idéias básicas para resolvê-los.

1.1 Afinal, O que é um Problema de Olimpı́ada?

Antes de responder esta pergunta, observe o seguinte exemplo.

Problema 1. Quatro garotos jogam tiro ao alvo. Cada um deles atirou três vezes. No alvo abaixo pode-se
ver os lugares atingidos. A pontuação é 6 para o centro e diminui um ponto para cada nı́vel mais distante.

Se os quatro empataram, determine:

(a) a pontuação total de cada jogador.

(b) a pontuação dos três tiros de cada jogador.

5
6 Prof. Bruno Holanda

Solução. A soma de todos os pontos obtidos foi 6 + 5 + 4 × 3 + 3 × 3 + 2 × 4 = 40. Como todos empataram,
cada um deve ter feito exatamente 10 pontos (isso responde o item a). Além disso é importante perceber
que ninguém errou nenhum dos tiros, já que há exatamente 12 dardos no alvo.

Note que um dos jogadores (digamos A) acertou um dos dardos no centro do alvo, fazendo 6 pontos. Para
completar os 10 pontos ele deve ter feito mais 4 pontos. Como é impossı́vel fazer apenas 1 ponto, ou dele ter
errado, só nos resta a possibilidade dele ter feito 2 pontos nos dois outros tiros.

[...] CONTINUE A EXPLICAÇÃO

Problema 2. (OBM 2002) O primeiro número de uma seqüência é 7. O próximo número é obtido da seguinte
maneira: Calculamos o quadrado do número anterior 72 = 49 e a seguir efetuamos a soma de seus algarismos
e adiconamos 1, isto é, o segundo número é 4 + 9 + 1 = 14. Repetimos este processo, obtendo 142 = 196 e
o terceiro número da seqüência é 1 + 9 + 6 + 1 = 17 e assim sucessivamente. Qual o 2002◦ elemento desta
seqüência?

Problema 3. Victor e Maria começam a trabalhar no mesmo dia. Victor trabalha 3 dias seguidos e depois
tem um dia de descanso. Maria trabalha 7 dias seguidos e descansa os outros 3. Quantos dias de descanso
em comum tiveram os dois durante os 1000 primeiros dias.

Problema 4. Complete a tabela abaixo de modo que:

i. A soma de quaisquer três vizinhos seja a mesma.

ii. A soma total dos números seja 171.

15 13

Problema 5. Em um hotel para cães e gatos 10% dos cães acham que são gatos e 10% dos gatos acham que
são cães. Verificou-se também que 20% de todos os hóspedes acham que são gatos e os 80% dos restantes
acham que são cães. Se no hotel existem 10 gatos quantos cães estão hospedados neste hotel?

Problema 6. São dadas 4 moedas aparentemente iguais. Sabe-se que uma delas é falsa (tem peso diferente
das demais e não se sabe se ela é mais leve ou mais pesada). Mostre como descobrir a moeda falsa com 2
pesagens em uma balançaa de dois pratos.

1.2 Organizando as Informações

Quando lemos um problema de matemática imediatamente podemos ver que ele está dividido em duas partes:
as informações e as perguntas. Você vai aprender, durante sua jornada como olı́mpico, que para resolver
um problema de matemática você deve conhecer várias técnicas. Uma das mais básicas é saber organizar as
informações que são oferecidas pelos problemas.
MÓDULO DE COMBINATÓRIA 7

Problema 7. (OCM 1990) A pesquisa realizada com as crianças de um conjunto habitacional, que apurou
as preferências em relação aos três programas de televisão: Alegre Amanhã (designado por A), Brincolândia
(designado por B) e Criança Feliz (designado por C) indicou os seguintes resultados:

Prog A B C AeB AeC BeC A,B e C Nenhum


Pref 100 150 200 20 30 40 10 130
Pergunta-se:

a) Quantas crianças foram consultadas?

b) Quantas crianças apreciam apenas um programa?

c) Quantas crianças apreciam mais de um programa?

Solução. Você deve ter percebido que existe um grande número de informações dadas. De certa forma,
essas informações já estão organizadas em uma tabela. Mas para resolver o problema vamos mudar nossa
representação, nosso ponto de vista. Vamos construir um diagrama de Venn, o popular diagrama de conjuntos:

C
140 130

20 30
10

60 10 100

A B

Podemos agora responder às perguntas facilmente:

a) Foram consultadas 10 + 10 + 20 + 30 + 60 + 100 + 140 + 130 = 500 crianças.

b) 60 + 100 + 140 = 300 crianças gostam de apenas um programa.

c) 10 + 10 + 20 + 30 = 70 crianças apreciam mais de um programa.

Problema 8. (OBM 1998) Encotre uma maneira de se escrever os algarismos de 1 a 9 em seqüência, de forma
que os números determinados por quaisquer dois algarismos consecutivos sejam divisı́veis por 7 ou por 13.

Uma idéia pouco inteligente seria listar todas as maneiras de se escrever os números de 1 a 9 e ver qual
(ou quais) delas possuem as propriedades pedidas. Por outro lado, podemos adaptar essa idéia para termos
8 Prof. Bruno Holanda

bem menos trabalho.

Solução. Primeiramente vamos listar todos os números de dois algarismos que são múltiplos de 7 ou 13. São
eles:
Múltiplos de 7: 14, 21, 28, 35, 42, 49, 56, 63, 70, 77, 84, 91, 98
Múltiplos de 13: 13, 26, 39, 52, 65, 78, 91

Como não podemos repetir nenhum algarismo, devemos descartar o 77. Por outro lado, nenhum dos
números acima (excluindo o 77) termina em 7. Daı́, pode-se ter certeza que o primeiro número da lista deve
ser 7. Para saber as possı́veis listas, usamos um diagrama de árvore:

== 5⊗
{{{
{{
{{{
{ // 9 // 1⊗
3>>
~~~
~
~~
~~
6 // 5⊗ == 5 // 6⊗
@@ {{
 {{
{
 {{
 {{
7 // 8 // 4
<<
// 2 // 1 // 3 // 9⊗
<<
<<
<<

// 1 // 3 // 5 // 2

// 6
9 BB
BB
BB
BB
B!!
6⊗

Representamos com um ⊗ quando não foi possı́vel continuar a lista sem repetir nenhum dı́gito. Assim, o
modo correto de se escrever os algarismo é: 784913526.

1.3 Tipos de Soluções

Existem três tipos básicos de solução: A prova direta, a prova inversa e a prova por absurdo. Vamos aprender
um pouco de cada uma nos próximos exemplos.

Problema 9. (Rússia 1997) Um polı́gono regular de 1997 lados é dividido em triângulos por diagonais que
não se cortam. Prove que exatamente um dos triângulos formados é acutângulo.

Solução. Como o polı́gono é regular, possui um circuncı́rculo. Dessa forma, o circuncentro dos triângulos
também é centro do 1997-ágono. Como 1997 é ı́mpar, o centro não está sobre nenhuma das diagonais. Logo,
está no interior de um único destes triângulos (e este será o único acutângulo).
MÓDULO DE COMBINATÓRIA 9

Problema 10. (Torneio das Cidades 2001) Podemos trocar um inteiro positivo n pelo produto a × b onde
a e b são inteiros positivos tais que a+b = n. Podemos obter 2001 a partir de 22, por uma seqüência de trocas?

Quando pensamos nesse problema, a primeira idéia que muitos têm é escrever 22 como soma de inteiros
positivos e começar a fazer a trocas. Mas em pouco tempo é possı́vel perceber que essa idéia não é muito
boa, pois logo no inı́cio o número de possı́veis trocas é muito grande e esse número aumenta à medida que
tentamos nos aproximar de 2001.

Dessa forma, devemos tentar resolver o problema de trás para frente, i.e., a partir de 2001 chegar em 22
através de trocas contrárias. Esse tipo de solução é conhecida na literatura inglesa como Working Backwards.
A tradução ao pé da letra seria “Trabalhando às Avessas”. Vejamos como funciona:

Solução. Note que 2001 = 3 × 667 pode ser obtido de 3 + 667 = 670, que pode ser obtido de 67 + 10 = 77 que
pode ser obtido de 7 + 11 = 18. Por outro, todo número n − 1 = (n − 1) × 1 pode ser obtido de (n − 1) + 1 = n.
Assim, basta seguir a seqüência abaixo:

22 → 21 → 20 → 19 → 18 → 77 → 670 → 2001.

Problema 11. Prove que em um grupo de 30 pessoas existem pelo menos duas delas com o mesmo número
de amigos.

Solução. Suponha o contrário, ou seja, que cada pessoa tem um número diferente de amigos. Note que
cada uma delas pode ter de 0 (quando ela não conhece ninguém) a 29 (quando ela conhece todos) amigos
no grupo. Por outro lado, de 0 a 29 (inclusive) existem exatamente 30 números inteiros. Como temos 30
pessoas, cada número estará assosciado a uma pessoa. Mas dessa forma, terı́amos uma pessoa que conhece
todos e outra que não conhece ninguém. Claramente isso não pode ocorrer. Temos assim um absurdo.

Este último problema foi um exemplo clássico como a prova por absurdo pode ser eficiente. Veja que a
fundamentação dela é relativamente fácil: Partimos de um fato que supomos verdadeiro, em seguida usando
argumentos lógicos provamos ser verdadeiro algo que é visivelmente falso. A única razão para isso ter ocor-
rido, deve-se ao fato de termos considerado verdadeiro uma hipótese falsa.

Problema 12. Ana, Bruna e Carla resolveram um total de 100 problemas do livro de matemática de modo
que cada uma delas resolveu separadamente 60 problemas. Chamamos um problema de difı́cil se ele foi
resolvido por somente uma das estudantes, e chamamos um problema de fácil se foi resolvido por todas.
Prove que o número total problemas difı́ceis excede a quantiodade de problemas fáceis em 20 unidades.

Problema 13. Um número natural n é dito azul se a soma de seu dı́gitos for igual à soma dos dı́gitos de
3n + 11. Prove que existem infinitos números azuis.

Problema 14. (OBM 2003) Considere as seqüências de inteiros positivos tais que cada termo mais a soma
dos seus algarismos é igual ao termo seguinte. Por exemplo: 6, 12, 15, 21, 24, 30, 33, 39 é uma seqüência
10 Prof. Bruno Holanda

nessas condições. Escreva a maior seqüência nessas condições cujo último termo é 103 e que satisfaz tais
condições.

Problema 15. (Torneio das Cidades 1987) 2000 maçãs estão distribuidas em vários potes. Podemos remover
potes e/ou remover algumas maçãs dos potes. Prove que é possı́vel ter um número igual de maçãs em cada
um dos potes restantes, com o número total de maças não menor do que 100.

Problema 16. Um quadrado é dividido em 100 retângulos por nove retas verticais e nove retas horizontais.
Sabe-se que, dentre esses retângulos exatamente nove são quadrados. Pode-se ter todos estes quadrados de
tamanhos difererentes?

Problema 17. Prove que 2 é irracional.
-2-
Contagem Simples e Permutações

De quantos modos podemos nos vestir? Quantos números menores que 1000 possuem todos os algarismos
pares? Contar coisas é algo tão antigo quanto a própria humanidade. Porém, ao longo do tempo as idéias
evoluiram e novas técnicas surgiram.

2.1 Contagem Direta

Vamos começar com um exemplo simples.

Problema 18. Uma porta só é aberta quando usamos simultaneamente a chave e o cartão corretos. Se você
possui duas chaves e três cartões, quantos testes devemos fazer para garantir que a porta irá abrir?

Solução. Podemos montar um diagrama (figura 2.1) para auxilar na solução do problema.

Figura 2.1: Abrindo uma Porta.

Note que para cada escolha de chave existem três maneiras para escolher o cartão. Como temos duas
chaves, o total de combinações é 2 × 3 = 6. Nesse caso, seriam necessários 6 testes para achar a combinação
correta.

O método que acabamos de usar para resolver o último problema é conhecido como princı́pio multiplica-
tivo. Nos próximos problemas vamos usá-lo de uma forma mais geral.

11
12 Prof. Bruno Holanda

Problema 19. Teddy possui 5 blusas, 3 calções e 2 pares sapatos. De quantas maneiras diferentes ele pode
se vestir?

Solução. Vamos primeiro contar o número de maneiras que Teddy pode escolher a blusa e a calça. Bem,
para cada calça que Teddy escolhe, ele tem ainda cinco maneiras de escolher a blusa. Como ele possui três
calças, o número total de modo de escolher o par (calça e blusa) é 5 × 3 = 15. Agora, para cada maneira de
escolher esse par, ele ainda tem duas maneiras de escolher os sapatos. Daı́, é fácil concluir que Teddy pode
se vestir de 5 × 3 × 2 = 30 maneiras diferentes.

Problema 20. De quantos modos podemos pintar um tabuleiro 1 × 4 usando apenas três cores, sem pintar
casas vizinhas da mesma cor?

Solução. Podemos pintar a primeira casa de três maneiras diferentes, a segunda de duas maneiras (não
podemos usar a cor da primeira casa), a terceira casa pode ser pintada de duas maneiras (não podemos usar
a cor da segunda casa), o mesmo ocorre com a quarta casa. Assim, o total de maneiras de pintar o tabuleiro
é 3 × 2 × 2 × 2 = 24.

Suponha que Carlos, Felipe, Marina e Ana estejam em uma fila. Se trocarmos a posição de alguns deles
dizemos que fizemos uma permutação. A pergunta é: Quantas permutações podemos ter usando quatro pes-
soas? Antes de resolver o problema vamos introduzir uma notação muito usada em problemas de contagem
por simplificar algumas contas.

Notação. Dado um número natural n, seja n! (leia n fatorial) o produto 1 · 2 · 3 · · · (n − 1) · n.

Observe que o conseito de fatorial está fortemente ligado à noção de permutação. Para fixar essa notação,
vamos resolver alguns exercı́cios simples:

1. Calcule 4!, 5! e 6!
100! 47!
2. Calcule e
98! 44!3!
3. Resolva a equação (m + 2)! = 72 · m!

4. Prove que
1 1 n
(a) − =
n! (n + 1)! (n + 1)!
(b) 2 · 4 · 6 · 8 · · · (2n) = 2n · n!

Problema 21. De quantas maneiras podemos formar uma fila com Carlos, Felipe, Marina e Ana?
MÓDULO DE COMBINATÓRIA 13

Solução. Podemos escolher o primeiro da fila de quatro maneiras, a segunda de três, a terceira de duas e a
última de apenas uma maneira (a pessoa que sobrar). Desse modo temos 4 · 3 · 2 · 1 = 4! permutações.

Problema 22. (OBM 2005) Num relógio digital, as horas são exibidas por meio de quatro algarismos. O
relógio varia das 00:00 às 23:00 horas. Quantas vezes por dia os quatro algarismos mostrados são todos pares?

Solução. Note que neste problema existe uma restrição nos dı́gitos que marcam as horas e no primeiro dı́tido
que marca os minutos. Dessa forma, em vez de pensar em cada dı́gito separadamente, vamos pensar em três
blocos de algarismos. O primeiro, que é formado pelos dois primeiros algarismos, pode assumir 7 valores
diferentes (00, 02, 04, 06, 08, 20 ou 22); o segundo é formado apenas pelo terceiro dı́gito e pode assumir 3
valores (0,2 ou 4); e o último dı́gito pode assumir 5 valores (0,2,4,6 ou 8). Logo, o total de vezes em que
todos aparecem pares é 7 × 3 × 5 = 105.

Agora vamos nos preocupar com alguns problemas mais “clássicos”. Apesar de serem problemas bem
conhecidos por todos, vamos abordá-los aqui, pois empregam idéias que são constantemente usadas em vários
problemas.

Problema 23. (Quantidade de Subconjuntos.) Quantos subconjuntos possui o conjunto M = {1, 2, 3, ..., 10}?

Solução. A cada subconjunto de M vamos associar uma seqüência de 10 dı́gitos que podem ser 0 ou 1. Essa
associação será dada através da seguinte regra: O primeiro termo dessa seqüência será 1 se o elemento 1
estiver no subconjunto e 0 caso contrário; O segundo termo dessa seqüência será 1 se o elemento 2 estiver no
subconjunto e 0 caso contrário; O terceiro termo dessa seqüência será 1 se o elemento 3 estiver no subconjunto
e 0 caso contrário; e assim por diante.
Por exemplo, o subconjunto {1, 2, 5, 8, 10} está associado à seqüência 1100100101, o subconjunto {2, 3, 5, 8}
está associado à seqüência 0110100100, enquanto o subconjuto vazio ∅ é representado por 0000000000. Note
que a quantidade de subconjuntos de M é igual à quantidade destas seqüências. Por outro lado, podemos
escolher cada dı́gito de duas formas e, conseqüêntimente, temos 210 seqüências (que é a mesma quantidade
de subconjuntos).

Problema 24. (Quantidade de Divisores) Seja n = pα1 1 · pα2 2 · · · pαk k um número natural na sua forma fatorada.
Então, n possui exatamente
(α1 + 1)(α2 + 1) · · · (αk + 1)

divisores inteiros positivos. Incluindo 1 e n.

Solução. Note que cada divisor positivo de n é da forma n = pβ1 1 · pβ2 2 · · · pβk k , onde cada expoente βi é um
número entre 0 e αi (inclusive). Dessa forma, temos (α1 + 1) maneiras de escolher o expoente de p1 ; (α2 + 1)
maneiras de escolher o expoente de p2 ; assim por diante. Logo, segue o resultado do princı́pio multiplicativo.

Problema 25. Numa sala existem 3 homens e 4 mulheres. De quantos modos é possı́vel selecionar um casal?
14 Prof. Bruno Holanda

Problema 26. Cada casa de um tabuleiro 2 × 2 pode ser pintado de verde ou amarelo. De quantas maneiras
podemos pintar o tabuleiro todo?
Problema 27. (OBM 2004) De quantos modos diferentes podemos pintar (usando apenas uma cor) as casas
de um tabuleiro 4 × 4 de modo que cada linha e cada coluna possua exatamente uma casa pintada?
Problema 28. Quantos números naturais de três algarismos distintos existem?
Problema 29. De quantos modos podemos por três torres de três cores diferentes em um tabuleiro 8 × 8 de
modo que nenhuma delas ataque outra?
Problema 30. De quantas maneiras podemos ir de A até B sobre a seguinte grade sem passar duas vezes
pelo mesmo local e sem mover-se para esquerda? A figura abaixo mostra um caminho possı́vel.

Problema 31. Ache a quantidade de números de três dı́gitos que não possuem dois algarismos consecutivos
iguais.
Problema 32. Responda os itens a seguir:
a) Quantos são os números de seis dı́gitos que não possuem dois algarismos consecutivos com a mesma
paridade?
b) Quantos são os números de seis dı́gitos distintos que não possuem dois algarismos consecutivos com a
mesma paridade?
Problema 33. Ache a quantidade númemos de cinco dı́gitos tais que toda seqüência de três algarismos
consecutivos é formada por elementos distintos.
Problema 34. (OBM 2005) Num tabuleiro quadrado 5 × 5, serão colocados três botões idênticos, cada um no
centro de uma casa, determinando um triângulo. De quantas maneiras podemos colocar os botões formando
um triângulo retângulo com catetos paralelos às bordas do tabuleiro?
Problema 35. Dizemos que a palavra algoritmo é um anagrama da palavra logaritmo pois é uma permutação
da letras de logaritmo. Sabendo disso, calcule a quantidade de anagramas da palavra vetor.
Problema 36. Quantos anagramas da palavra vetor termina em uma vogal?
Problema 37. De quantas maneiras é possı́vel colocar em uma prateleira 5 livros de matemática, 3 de fı́sica
e 2 de biologia, de modo que livros de um mesmo assunto permaneçam juntos?
Problema 38. Quantos anagramas da palavra vetor possuem as vogais separadas?
Problema 39. Uma embarcação deve ser tripulada por oito homens, dois dos quais só remam do lado direito
e um apenas do lado esquerdo. Determine de quantos modos esta tripulação pode ser formada, se de cada
lado deve haver quatro homens.
Obs : A ordem dos homens deve ser considerada.
MÓDULO DE COMBINATÓRIA 15

Problema 40. De quantas formas podemos colocar 4 bolas verdes e 4 bolas amarelas em um tabuleiro 4 × 4
de modo que cada coluna e cada linha possua exatamente uma bola de cada cor.

Problema 41. Responda os itens a seguir:


a) Ache a quantidade de divisores positivos de 3600.
b) Quantos desses divisores são pares?
c) Quantos são quadrados perfeitos?

Problema 42. (Maio 2006) Um calendário digital exibe a data: dia, mês e ano, com 2 dı́gitos para o dia, 2
dı́gitos para o mês e 2 dı́gitos para o ano. Por exemplo, 01-01-01 corresponde a primeiro de janairo de 2001 e
25-05-23 corresponde a 25 de maio de 2023. Em frente ao calendário há um espelho. Os dı́gitos do calendário
são como os da figura abaixo.

Se 0, 1, 2, 5 e 8 se reflentem, respectivamente, em 0, 1, 5, 2 e 8, e os outros dı́gitos perdem sentido ao se


refletirem, determine quantos dias do século, ao se refletirem no espelho, correspondem também a uma data.

Problema 43. Quantas são as triplas (A, B, C) de subconjuntos de M = {1, 2, ..., 100} tais que A∪B ∪C = M
e A ∩ B ∩ C = φ.

Problema 44. Um número natural n é dito elegante se pode ser escrito como soma de cubo com um quadrado
(n = a3 + b2 , onde a, b ∈ N). Entre 1 e 1000000 existem mais números que são elegantes ou que não são?

2.2 Separando em Casos

Uma técnica bastante usada em problemas de contagem é a idéia de separar o problema em casos disjuntos.
Fazemos isso para evitar contar várias vezes a mesma configuração, ou esquecer algumas delas.

Problema 45. O alfabeto da Tanzunlândia é formado por apenas três letras: A, B e C. Uma palavra na
Tanzunlândia é uma seqüência com no máximo 4 letras. Quantas palavras existem neste paı́s?

Solução. Existem 3 palavras com uma letra, 32 com duas letras, 33 com três letras, e 34 com quatro letras.
Logo, o total de palavras é 3 + 32 + 33 + 34 = 120.

Problema 46. De quantos modos podemos pintar (usando uma de quatro cores) as casas da figura a baixo
de modo que as casas vizinhas tenham cores diferentes?

1 2

4 3

Solução. Vamos separar o problema em dois casos:


16 Prof. Bruno Holanda

i. Se as casas 1 e 3 tiverem a mesma cor, temos quatro maneiras de escolher essa cor. Podemos escolher
a cor da casa 2 de três maneiras (basta não ser a cor usadas nas casas 1 e 3), o mesmo vale para casa
4. Logo, temos 4 × 3 × 3 = 36 maneiras de pintar dessa forma.

ii. Agora se 1 e 3 têm cores diferentes, podemos escolher a cor da casa 1 de quatro maneiras, da casa 3 de
três maneiras e, das casas 2 e 4, podemos escolher de duas maneiras cada. Assim, temos 4×3×2×2 = 48
maneiras de pintar desta outra forma.
Desse modo, podemos concluir que existem 36 + 48 = 84 maneiras de pintar a rosquinha.

Problema 47. (AHSME 1998) Um número de telefone d1 d2 d3 − d4 d5 d6 d7 é dito “memorável” se a seqüência


d1 d2 d3 é igual a uma das seqüências d4 d5 d6 ou d5 d6 d7 (possivelmente ambas). Ache o número de telefones
memoráveis.

Solução.
Problema 48. Escrevem-se todos os inteiros de 1 a 9999. Quantos números têm pelo menos um zero?
Problema 49. Quantos números de três dı́gitos possuem todos os seus algarismos com a mesma paridade?
Problema 50. Quantos são os números de quatro algarismos que possui pelo menos um dı́gito repetido?
Problema 51. De quantas maneiras podemos colocar um rei preto e um rei branco em um tabuleiro de xadrez
(8 × 8) sem que nenhum deles ataque o outro?
Problema 52. Quantos são os naturais pares que se escrevem com três algarismos distintos?
Problema 53. Na cidade Gótica as placas das motos consistem de três letras. A primeira letra deve estar no
conjunto {C, H, L, P, R}, a segunda letra no conjunto {A, I, O}, e a terceira letra no conjunto {D, M, N, T }.
Certo dia, decidiu-se aumentar o número de placas usando duas novas letras J e K. O intendente dos
transportes ordenou que as novas letras fossem postas em conjuntos diferentes. Determine com qual opção
podemos obter o maior número de placas.
Problema 54. Um número N de sete dı́gitos é dito um ancestral se for possı́vel cortar um de seus dı́gitos e
obter o número 123456. Quantos números ancestrais existem?
Problema 55. (Maio 1998) Cada um dos seis segmentos da figura abaixo deve ser pintado de uma de quatro
cores de modo que segmentos vizinhos não tenham a mesma cor. De quantas maneiras podemos fazer isso?

Problema 56. Um prova de matemática é composta de 7 problemas. Em cada problemas pode-se obter 0,
1, 2, 3 ou 4 pontos. De quantas maneiras diferentes podemos ter uma pontuação total igual a 24?
Problema 57. (Banco Cone Sul) Um número de três dı́gitos é dito equilibrado, se um dos seu dı́gitos é a
media aritmética dos outros dois. Quantos são os números equilibrados?
MÓDULO DE COMBINATÓRIA 17

Problema 58. Tenho 10 livros distintos de matemática, 3 dos quais são vermelhos. De quantos modos posso
ordená-los em uma prateleira de modo que não existam dois livros vermelhos juntos?

Problema 59. De quantos modos podemos pintar exatamente 9 casas de um tabuleiro 10 × 2 de modo que
não existam duas casas vizinhas pintadas.

2.3 Contagens Múltiplas

O próprio nome da seção nos dá uma dica sobre que tema vamos falar a partir de agora.

Problema 60. Em uma festa havia 6 homens e 4 mulheres. De quantos modos podemos formar 3 pares como
essas pessoas?

Problema 61. De quantas formas podemos por oito pessoas em uma fila se Alice e Bob devem estar juntos,
e Carol deve estar em algum lugar atrás de Daniel?

Problema 62. Quantas diagonais possui um dodecágono regular?

Problema 63. De quantas maneiras podemos por três torres de mesma cor em um tabuleiro 8 × 8 de modo
que nenhuma delas ataque a outra?

Problema 64. Quantas triplas distintas podemos formar a partir de um grupo de sete pessoas?

Problema 65. Quantos anagramas possui a palavra matematica (desconsidere o acento)?

Problema 66. Temos um grupo de cinco garotos e seis garotas e queremos escolher quatro casais para
dançarem ao mesmo tempo em um baile. De quantos modos podemos fazer esta escolha?

Problema 67. (OBM 2003) Num tabuleiro mostrado a seguir, escrevemos números inteiros de 1 a 9 obede-
cendo a seguinte regra: A > B, C > D, A > C e B > D

A B
C D

(a) Quantos tabuleiros diferentes existem tais que B = C?

(b) Quantos tabuleiros diferentes existem no total?

Problema 68. De quantas maneiras podemos pintar as faces de cubo usando dez cores de modo que cada
face fique com uma cor diferente.

Problema 69. (AIME 1996) Duas casas de um tabuleiro 7 × 7 são pintadas de amarelo e as outras são
pintadas de verde. Duas pinturas são ditas equivalentes se uma é obtida a partir de uma rotação aplicada no
plano do tabuleiro. Quantas pinturas inequivalentes existem?

Problema 70. Em uma sala de aula existem x meninas e y meninos. De quantas formas eles podem ficar
em uma fila, se as meninas devem ficar em ordem crescente de peso, e os meninos também? (Suponha que 2
pessoas quaisquer não tenham o mesmo peso.)
18 Prof. Bruno Holanda

Problema 71. Considere um torneio de xadrez com 10 participantes. Na primeira rodada cada participante
joga somente uma vez, de modo que há 5 jogos realizados simultaneamente. De quantas maneiras esta
primeira rodada pode ser realizada?

Problema 72. Doze cavaleiros estão sentados em torno de uma mesa redonda. Cada um dos 12 cavaleiros
considera seus dois vizinhos como rivais. Deseja-se formar um grupo de 5 cavaleiros para salvar uma princesa.
Nesse grupo não poderá haver cavaleiros rivais. Determine de quantas maneiras é possı́vel escolher esse grupo.

Problema 73. (Rioplatense 1999) De quantas maneiras podemos pintar as casas de um tabuleiro 2×2 usando
sete cores?
Obs: Duas pinturas são consideradas iguais se uma pode ser obtida aplicando uma rotação na outra.

2.4 Números Binomiais

Esta seção pode ser considerada como uma continuação da anterior. Aqui vamos estudar um tipo bem
especial de contagem múltipla: as combinações.

Problema 74. Em uma classe com 10 alunos deseja-se escolher três para formar um grêmio estudantil. De
quantos modos podemos fazer isto?

Solução. Bem, vamos começar tentando usar as idéias que aprendemos nas seções anteriores. Como são 10
alunos, temos 10 maneiras para escolher a primeira pessoa, 9 para escolher a segunda e 8 para a terceira.
Por outro lado, a ordem em que elas são escolhidas não é importante. Como podemos permutar um grupo
de 3 pessoas de 3 × 2 × 1 = 6 maneiras, cada grupo foi contado 6 vezes. Logo, o número total de grêmios que
10 × 9 × 8
podemos formar é = 120.
6
De modo mais geral, podemos contar o número de maneiras de se escolher n elementos de um grupo de
m objetos. Por ser um tipo especial de contagem que frequentemente aparece nos problemas de matemática,
ela recebeu uma simbologia própria. Dessa forma, o número de maneiras Smn de se escolher n objetos de um
grupo de m destes é, por definição:  
m m!
Smn := =
n n!(m − n)!
Já a segunda parte da igualdade é dada por uma contagem análoga a que fizemos no problema anterior.

Imagine que você tenha alguns objetos e deseja distribuı́-los entre alguns de seus amigos. A pergunta
é: De quantos modos podemos fazer isso? Claramente a resposta deve depender do número de objetos e de
amigos. Problemas como esse são conhecidos como problemas de distribuição.

Problema 75. Etevaldo possui três filhos e dez moedas iguais. Ele deseja distribuir essas moedas entre seus
filhos de modo que cada um deles receba pelo menos uma moeda. De quantos modos ele pode fazer isso?

Solução. Para resolver esse problema vamos observar o seguinte diagrama: Suponha que Etevaldo colocou
as moedas sobre a mesa. Para decidir quantas moedas cada filho vai receber, ele vai separar as moedas
colocando dois palitos de picolé em dois dos espaços que existem entre duas moedas vizinhas. Esses palitos
irão separar as moedas em três partes. E cada uma dessas partes corresponderá à quantidade de moedas
MÓDULO DE COMBINATÓRIA 19

Figura 2.2: Distribuindo Moedas.

de cada filho irá receber. Por exemplo, o diagrama indicado na figura 2.3 representa a distribuição onde o
primeiro filho recebe três moedas, o segundo duas e o terceiro cinco. Agora, como temos nove espaços entre

Figura 2.3: Distribuindo Moedas.


9

as moedas e dois palitos, temos um total de 2 = 36 maneiras de fazer a distribuição.

Você deve ter percebido que no problema anterior usamos uma idéia bem interessante: Não atacamos o
problema diretamente, usamos um modelo que se ajustava perfeitamente às condições do problema. (Lembre-
se que já usamos uma técnica similar para resolver o problema 23). Esse tipo de idéia é conhecido como
Princı́pio Bijetivo e é uma das ferramentas mais usadas para resolver problemas de combinatória. Vamos
aplicá-la novamente para resolver o próximo problema.

Problema 76. O mapa de uma cidade é representado pela figura 2.4. Todas as ruas são de mão única, de
tal modo que uma pessoa só pode dirigir para o norte ou para o leste. Quantas rotas diferentes podemos
selecionar para ir de P até Q?

Figura 2.4: Contando Rotas.

Solução. Inicialmente, veja que para ir de P até Q, devemos ir para o norte 5 vezes e para o leste 7 vezes.
Desse modo, vamos associar cada uma das rotas a uma seqüência de 12 letras, 5 iguais a N e as outra iguais
a L. Faremos essa associação da seguinte forma: Se a primeira letra for N , então significa que o primeiro
movimento foi para o norte, se a segunda letra for L, então significa que o segundo movimento foi para o leste
e assim por diante. Por exemplo, na figura 2.4 a rota destacada corresponde à seqüência LN N N LLLLN LLN .

Agora, para achar o número de rotas basta contar o número destas seqüências. Como  cada seqüência
12
possui 12 letras, onde exatamente 5 delas são N e as demais são L, temos um total de 5 destas.
20 Prof. Bruno Holanda

Problema 77. Um estudante possui 7 figurinhas, e um amigo seu possui 8. De quantos modos o primeiro
aluno pode trocar 3 de suas figurinhas por 3 figuras de seu amigo, sabendo que todas as figuras são diferentes?

Problema 78. Em um clube de xadrez participam 2 garotas e 7 garotos. Devemos montar um time com
quatro pessoas, com pelo menos uma menina. De quantos modos podemos fazer isto?

Problema 79. Uma pessoa possui 6 amigos. A cada noite, durante 5 noites, ela convidará 3 de seus amigos
para um jantar, de modo que cada grupo nunca seja convidado mais de uma vez. De quantas maneiras ela
poderá fazer isto?

Problema 80. Quantos triângulos podemos formar usando os pontos marcados na figura 2.5?

Figura 2.5: Contando Triângulos.

Problema 81. (OBM 2004) Os doze alunos de uma turma de olimpı́ada saı́am para jogar futebol todos os
dias após a aula de matemática, formando dois times de 6 jogadores cada e jogando entre si. A cada dia eles
formavam dois times diferentes dos times formados em dias anteriores. Ao final do ano, eles verificaram que
cda 5 alunos haviam jogado juntos num mesmo time exatamente uma vez. Quantos times diferentes foram
formados ao longo do ano?
-3-
Jogos

Jogos sempre foi um tema que chamava muito a atenção das pessoas. Podemos fazer uma lista enorme de
jogos famosos como o xadrez, as damas e os jogos com baralho. Porém, todos eles têm um “problema”: para
vencê-los é necessário um treinamento árduo ou uma boa dose de sorte.

Mas, e se existisse algum tipo de jogo em que você pudesse ganhar sempre, independente de como seu
adversário jogasse? Seria uma boa, não?! Pois esses jogos existem e são um dos assuntos mais abordados em
provas de olimpı́ada. Neste capı́tulo vamos mostrar vários destes jogos e as principais estratégias vencedoras:
a simetria e o uso das posições vencedoras.

3.1 Simetria

Uma das estratégias mais simples é o uso de alguma simetria que pode ocorrer durante o jogo em vantagem
de um dos jogadores, forçando sempre uma nova rodada para o jogador “destinado à derrota”. Para entender
melhor veja o seguinte exemplo:

Problema 82. Pedro e Mônica jogam em um tabuleiro 1 × 11. Cada um, em sua vez, pode pintar um dos
quadrados (que não foram pintados anteriormente), ou dois quadrados consecutivos (se ambos estiverem
brancos). Quem não puder mais jogar perde. Sabe-se que Pedro será o primeiro a jogar. Quem pode sempre
garantir a vitória?

Solução. Pedro sempre poderá ganhar se seguir a seguinte estratégia:

(i) Inicialmente, Pedro deve pintar o quadrado do meio.

(ii) Agora, depois que Mônica fizer sua jogada, Pedro deve jogar sempre simetricamente em relação ao
centro do tabuleiro (i.e. sempre deixando o tabuleiro simétrico). Por exemplo, se Mônica jogar nas
casas 9 e 10, Pedro deve jogar nas casas 2 e 3.

21
22 Prof. Bruno Holanda

× × × z z

(iii) Assim, Mônica nunca poderá ganhar, pois na sua jogada ela “quebra a simetria” e a configuração final
do jogo todas as casas estarão pintadas, ou seja, a configuração é simétrica.

O próximo exemplo é um dos problemas que apareceu na prova da OBM de 2004. Vamos apresentar uma
solução diferente da solução proposta na Eureka! 22, usando simetria:

Problema 83. Arnaldo e Bernardo disputam um jogo em um tabuleiro 2 × n :

As peças do jogo são dominós 2 × 1. Inicialmente Arnaldo coloca um dominó cobrindo exatamente duas
casas do tabuleiro, na horizontal ou na vertical. Os jogadores se revezam colocando uma peça no tabuleiro,
na horizontal ou na vertical, sempre cobrindo exatamente duas casas do tabuleiro. Não é permitido colocar
uma peça sobre outra já colocada anteriormente. Quem não conseguir colocar uma peça no tabuleiro perde.

Qual dos dois jogadores tem uma estratégia vencedora, ou seja, uma estratégia que o leva à vitória quais-
quer que sejam as jogadas de seu adversário, para:
a) n = 2004?
b) n = 2005?

Solução. Quando n = 2005 o primeiro jogador garante a vitória. Ele pode fazer isto colocando um dominó
na vertical no meio do tabuleiro e, em seguida, jogar simetricamente ao segundo jogador. Quando n = 2004
o tabuleiro possui um número par de colunas. Desse modo, o segundo ganha jogando simetricamente ao
primeiro jogador.

Como você deve ter visto, usar a simetria é realmente uma técnica muito eficiente. Porém, às vezes,
usar apenas a simetria não é suficiente para resolver o problema. Observe o próximo exemplo retirado da
olı́mpiada da Bielorússia de 2000.

Problema 84. Tom e Jerry jogam o seguinte jogo. Eles colocam alternadamente pinos idênticos em casas
vazias de um tabuleiro 20 × 20 (um pino de cada vez). Tom é o primeiro a jogar. Vence quem, em sua jogada,
formar um bloco de quatro pinos vizinhos. Dois pinos são vizinhos se estiverem em casas com um lado em
comum. Determine quem possui a estratégia vencedora.

Solução.
MÓDULO DE COMBINATÓRIA 23

Problema 85. Sobre uma mesa existem duas pilhas (uma com 15 e outra com 16 pedras). Em um jogo cada
jogador pode, em sua vez, retirar qualquer quantidade de pedras de apenas uma pilha. Quem não puder
mais jogar perde. Quem possui a estratégia vencedora?
Problema 86. Dois jogadores colocam alternadamente bispos (da mesma cor) em um tabuleiro 8 × 8, de
forma que nenhum bispo ataque outro. Quem não puder mais jogar perde.
Problema 87. Dois jogadores colocam alternadamente reis (da mesma cor) em um tabuleiro 9 × 9, de forma
que nenhum rei ataque outro. Quem não puder mais jogar perde.
Problema 88. São dados um tabuleiro de xadrez (8 × 8) e palitinhos do tamanho dos lados das casas do
tabuleiro. Dois jogadores jogam alternadamente e, em cada rodada, um dos jogadores coloca um palitinho
sobre um lado de uma das casas do tabuleiro, sendo proibido sobrepor os palitinhos.
Vence o jogador que conseguir completar primeiro um quadrado 1 × 1 de palitinhos. Supondo que nenhum
dos jogadores cometa erros, qual dos dois tem a estratégia vencedora?
Problema 89. São dados vinte pontos ao redor de um cı́rculo. Cada jogador em sua vez pode ligar dois
desses pontos se essa novo segmento não cortar os feitos anteriormente. Quem não puder mais traçar nenhum
segmento perde.
Problema 90. Dois jogadores colocam alternadamente x’s e o’s em um tabuleiro 9×9. O primeiro escreve x’s
e o segundo o’s. Quando o tabuleiro for completamente preenchido o jogo termina e os pontos são contados.
Um ponto é dado ao jogador para cada linha ou coluna em que ele possuir mais casas dos que o adversário.
O jogador que possuir mais pontos vence. Quem pode sempre ganhar?
Problema 91. Um pino está no centro de um tabuleiro 11 × 11. Dois jogadores movem alternadamente o
pino para qualquer outra casa do tabuleiro, mas a cada movimento (a partir do segundo) deve ser maior que
o anterior. O jogador que não puder mais jogar perde. Ache a estratégia vencedora.
Problema 92. Um jogo consiste em quebrar um tabuleiro 5 × 10 ao longo de suas linhas. Ganha o primeiro
jogador que obter um quadrado 1 × 1. Quem tem a estratégia vencedora?
Problema 93. (Russia 1997) Os números 1, 2, 3, ..., 1000 são escritos no quadro. Dois jogadores apagam
alternadamente um dos números da lista até que só restem dois números. Se a soma desses números for
divisı́vel por 3, o primeiro jogador vence, caso contrário vence o segundo. Quem tem a estratégia vencedora?

3.2 Posições Vencedoras

Alguns tipos de jogos possuem certas configurações que sempre levam um jogador à vitória. Essas con-
figurações são chamadas de posições vencedoras. O próximo exemplo é um jogo bastante simples em que essa
estratégia aparece facilmente.
Problema 94. Na primeira casa de um tabuleiro 1 × 13 está uma moeda. Tiago e Maria movem a moeda
alternadamente. Em cada turno é permitido avançar 1, 2, 3, 4 ou 5 casas. Quem colocar a moeda na última
casa é o vencedor. Se Maria começar jogando, ela pode ter certeza da vitória?

Solução. Como em muitos problemas de olimpı́ada, vamos analisar alguns casos pequenos. Vamos supor
que em vez de 13 casas o tabuleiro tivesse apenas quatro. Neste caso, fica fácil ver que quem começa ganha
24 Prof. Bruno Holanda

basta avançar três casas.


O mesmo iria ocorrer se o tabuleiro tivesse 2, 3, 4, 5 ou 6 casas. Porém, em um tabuleiro 7 × 1 o primeiro
jogador perde. Veja que após a primeira jogada a moeda estará em uma das casas 2, 3, 4, 5 ou 6. E já sabemos
que essas casas levam o jogador à vitória.

Desse modo, vamos dizer que 7 é uma posição perdedora e 6, 5, 4, 3 e 2 são posições vencedoras. Assim, se
um o jogador estiver em uma das casas 8, 9, 10, 11 ou 12, ele pode garantir a vitória movendo a moeda para
a casa 7, deixando o seu adversário em uma posição perdedora. Com isso, podemos afirmar que as posições
8, 9, 10, 11 e 12 também são posições vencedoras.
Resta analisar a 13a casa. Observe que a partir desta casa podemos mover a moeda apenas para uma das
casas 8, 9, 10, 11 ou 12 que são vencedoras. Daı́, quem começar perde pelo simples fato de iniciar em uma
posição perdedora.

A grande dificuldade para a maioria dos alunos é descobrir quais são as posições vencedoras de um jogo.
Para evitar esse tipo de problema, tenha sempre em mente as seguintes definições:

(a) Posição vencedora: A partir dela, podemos escolher um movimento e repassar uma posição
perdedora para o adversário.

(b) Posição perdedora: A partir dela, é impossı́vel escolher um movimento e repassar uma
posição perdedora para o adversário. Ou seja, não importa o movimento escolhido, o adversário
irá receber uma posição vencedora.

E como fazer para descobrir quais são as posições vencedoras e perdedoras? A melhor maneira de se
fazer isto é analisando o final do jogo e aplicar as definições acima. Vamos praticar um pouco resolvendo o
próximo problema.

Problema 95. Em um tabuleiro 8 × 8, uma torre está na casa a1. Dois jogadores movem a torre com objetivo
de colocar a torre na casa h8. Sabendo que a torre pode mover-se apenas para cima ou para direita (quan-
tas casas o jogador desejar) e que não pode-se passar a vez, determine qual jogador tem a estratégia vencedora.

Solução. Primeiramente note que todas as casas da última linha e da última coluna (exceto a h8) são
vencedoras pois, a partir delas podemos escolher um movimento que nos leve à vitória. Com, isso a casa g7
se torna perdedora pois, a partir dela qualquer movimento leva o outro jogador a uma posição vencedora
(veja a figura 1).
MÓDULO DE COMBINATÓRIA 25

Figura 1 Figura 2 Figura 3

Agora, como g7 é perdedora, as demais casas da sétima linha e da sétima coluna são vencedoras. Mais
ainda, a casa f 6 também deve ser perdedora (figura 2). Continuando de maneira análoga, obtemos que a
casa a1 é perdedora (figura 3). Logo, quem começar, perde.

Problema 96. Sobre uma mesa existem duas pilhas de moedas com 11 moedas cada. Em cada turno, um
jogador pode retirar duas moedas de uma das pilhas ou retirar uma moeda de cada pilha. O jogador que
não puder mais fazer movimentos perde.

Problema 97. Tom e Jerry jogam um jogo e Tom faz a primeiro passo. Em cada turno o jogador pode
diminuir de um dado natural N um dos seus dı́gitos não-nulos. Inicialmente o número N é 1234. O jogador
que obter zero ganha. Quem pode garantir a vitória?

Problema 98. Uma pilha de 500 pedras é dada. Dois jogadores jogam o seguinte jogo: Em cada turno, o
jogador pode retirar 1, 2, 4, 8, ... (qualquer potência de 2) pedras da pilha. O jogador que não puder mais
jogar perde.

Problema 99. Em uma caixa existem 300 bolinhas. Cada jogador pode retirar não mais do que a metade
das bolinhas que estão na caixa. O jogador que não puder mais jogar perde.

Problema 100. Sobre uma mesa existem duas pilhas (uma com 7 e outra com 15 pedras). Em um jogo
cada jogador pode, em sua vez, retirar qualquer quantidade de pedras de apenas uma pilha ou a mesma
quantidade de ambas as pinhas. Quem não puder mais jogar perde. Quem possui a estratégia vencedora?

Problema 101. Sobre uma mesa existem duas pilhas (cada uma com 11 pedras). Em um jogo cada jogador
deve retirar duas pedras de uma pilha e uma da outra. O jogador que não puder mais jogar perde. Quem
possui a estratégia vencedora?

3.3 Idéias Avançadas

Problema 102. (Rússia 1996) Dois jogadores disputam um jogo em um tabuleiro 100 × 100. O primeiro
jogador escolhe um uma casa livre do tabuleiro e marca um ×. Em seguida, o segundo jogador deve colocar
um dominó 1 × 2 cobrindo duas casas livres, uma das quais seja marcada. O primeiro jogador vence se
todo o tabuleiro for coberto, caso contrário o segundo jogador vence. Qual dos jogadores possui a estratégia
vencedora?
-4-
Casa dos Pombos

O princı́pio da casa dos pombos também é conhecido em alguns paı́ses (na Rússia, por exemplo) como
Princı́pio de Dirichlet pois, foi o matemático Lejeune Dirichlet o primeiro matemático a usa este método
para resolver problemas não triviais. Outros matemáticos que se destacaram por usarem essa idéia para
resolver diversos problemas foram os húngaros Erdős e Szekeres. Vamos abordar este princı́pio da seguinte
maneira:

“Se em n caixas são postos n + 1 pombos, então pelo menos uma


caixa terá mais de um pombo.”

Alguns Exemplos:
i. Em um grupo de três pessoas, pelo menos duas delas são do mesmo sexo.
ii. Em um grupo de 13 pessoas, pelo menos duas delas têm o mesmo signo.
iii. Em um grupo de 5 cartas de baralho, pelo menos duas são do mesmo naipe.
iv. Na cidade de Fortaleza, existem pelo menos duas pessoas com o mesmo número de fios de cabelo.
Agora vamos ver como algo tão simples pode resolver problemas aparentemente difı́ceis:

Problema 103. Escolhem-se 5 pontos ao acaso sobre a superfı́cie de um√quadrado de lado 2. Mostre que pelo
menos dois deste pontos estão em um distância menor que ou igual a 2.

Solução. Divida o quadrado em quatro quadrados menores como na figura ao lado. Como temos cinco pontos
e quatro quadrados, teremos pelo menos dois pontos no mesmo quadradinho. Como a maior distância entre
dois pontos do mesmo quadradinho não supera a medida de sua diagonal, o resultado segue de imediato.

√ 2

26
27

Problema 104. Nove pontos são postos sobre a superfı́cie de um tetraedro regular com 1cm de aresta. Prove
que detre esses pontos é possı́vel acahr dois com distância (espacial) não maior que 0.5cm.

Solução. Vamos particionar a superfı́cie do tetraedro em 16 triângulos eqüiláteros congruentes, dividindo


cada face em quatro partes usando suas bases médias. Agora vamos criar 8 regiões pitando esses triângulos
de acordo com a seguinte regra: os triângulos que possuem um mesmo vértice do tetraedro serão pintados
da mesma cor; dessa forma já usamos quatro cores diferentes para 12 triângulos e os outros quatro vamos
pintar usando as demais cores. De acordo com o Princı́pio da Casa dos Pombos, pelo menos dois dos nove
pontos estarão na mesma região. Fica apenas faltando que a distância máxima entre dois pontos da mesma
região é no máximo 0.5cm.

Como acabamos de ver, usar o princı́pio da casas dos pombos não é difı́cil. O difı́cil está em achar o que
serão nossos “pombos” e “caixas”. O próximo problema é, a priori, um problema de teoria dos números.
Porém, vamos usar o princı́pio da casa dos pombos para resolvê-lo.

Problema 105. Prove que dados sete inteiros positivos, existem dois cuja soma ou a diferença é um múltiplo
de 10.

Solução. Vamos montar seis caixas C0 , C2 , ..., C5 onde um inteiro está na caixa Ci se é congruente a i ou a
−i módulo 10. Sabemos que existirão dois inteiros na mesma caixa. Dessa forma, se eles forem incongruentes
módulo 10, basta somá-los. Caso contrário, faça a sua diferença.

Problema 106. Cada casa de um tabuleiro 3 × 7 é pintado de preto ou branco. Mostre que é possı́vel achar
um retângulo (com lados paralelos aos do tabuleiro) cujas quatro pontos são da mesma cor.

Solução. Cada coluna deste tabuleiro pode ser pintado de uma das seguintes formas:

1 2 3 4 5 6 7 8
28 Prof. Bruno Holanda

Observe que se a pintura 1 for escolhida, bastaria uma coluna do tipo 2, 3 ou 4 para formar um retângulo.
Com isso, nos restariam apenas mais quatro outras pinturas porém, temos sete colunas. Daı́, pelo principio
da casa dos pombos terı́amos duas colunas iguais. O mesmo ocorre com a coluna do tipo 8.
Agora suponha que nenhuma das colunas for do tipo 1 ou 8. Dessa forma, restaria apenas 6 tipos de
pinturas. Assim, pelo princı́pio da casas dos pombos, duas delas seriam iguais.

Problema 107. (Teorema de Ramsey) Em um grupo de seis pessoas sempre existem três que se conhecem
mutuamente ou três que não se conhecem mutuamente.

Prova. Para resolver este problema vamos usar a linguagem dos grafos. Dessa forma, pense em um grafo com
seis vértices A, B, C, D, E, F . Uma aresta contı́nua irá representar uma “amizade” e uma aresta pontilhada,
uma “inimizade”. Fixado o vértice A, sabemos que ele possui cinco arestas. Como só há dois tipos de aresta,
um dos tipos foi usado pelo menos três vezes. Sem perca de generalidade, suponha que o tipo “continua” foi
escolhido três vezes.

F
B
E

C D

Agora, se uma das arestas BC,CD ou DB for contı́nua, teremos três pessoas se conhecendo mutuamente.
Caso contrário, as três são pontilhadas. Neste caso, B,C e D não se conhecem mutuamente.

Problema 108. Cinquenta e um pontos são postos no interior de um quadrado de lado 1 metro. Prove que
existe um conjunto de três desses pontos podem ser cobertos por um quadrado de lado 20 centı́metros.

Problema 109. Em cada casa de um tabuleiro 3 × 3 é colocado um dos números −1, 0, 1. Prove que, dentre
as oito somas ao longo de uma mesma linha, coluna ou diagonal, existem duas iguais.

Problema 110. Prove que de qualquer conjunto de dez inteiros podemos escolher um subconjunto cuja soma
é um múltiplo de 10.

Problema 111. Prove que existe uma potência de 3 terminada nos dı́gitos 001 (na base decimal).

Problema 112. Mostre que um triângulo eqüilátero não pode ser totalmente coberto por outros dois triângulos
eqüiláteros menores.

Problema 113. Dados 5 pontos no plano com coordenadas inteiras, prove que pelo menos um dos dez pontos
médio gerados por eles também possui coordenadas inteiras.

Problema 114. O plano é pintado usando duas cores. Prove que existem dois pontos de mesma cor distando
exatamente um metro.
29

Problema 115. (Putnam) O plano é pintado usando três cores. Prove que existem dois pontos de mesma
cor distando exatamente um metro.
Problema 116. O plano é totalmente pintado usando duas cores. Prove que existe um retângulo cujos vértices
são todos da mesma cor.
Problema 117. (Bielorussia 1996) Em um grupo de 29 hobbits existem alguns deles que falam a verdade e
os outros que sempre mentem. Em um certo dia de primavera, todos eles se sentaram ao redor de uma mesa,
e cada um deles falou que seus dois vizinhos eram mentirosos.
a) Prove que pelo menos 10 hobbtis falavam a verdade.
b) É possı́vel que exatamente 10 deles falem a verdade?
Problema 118. Em cada casa de um tabuleiro 10 × 10 é posto um inteiro de modo que a diferença positiva
entre os inteiros de duas casas vizinhas (lado em comum) é no máximo 5. Prove que dois destes inteiros
devem ser iguais.
Problema 119. Trinta e três torres são postas em um tabuleiro 8 × 8. Prove que podemos escolher cinco
delas sem que nenhuma ataque a outra.
Problema 120. Em uma sapataria existem 200 botas de tamanho 41, 200 botas de tamanho 42, e 200 botas
de tamanho 43. Dessas 600 botas, 300 são para o pé esquerdo e 300 para o direito. Prove que existem pelo
menos 100 pares de botas usáveis.
Problema 121. Onze estudantes formaram cinco grupos de estudo. Prove que existem dois alunos A e B,
tais que em todo grupo que inclui A também inclui B.
Problema 122. (TT 1994) Existem 20 alunos em uma escola. Quaisquer dois deles possui um avó em comum.
Prove que pelo menos 14 deles possui um avó em comum.
Problema 123. (Rússia 1997) Uma sala de aula possui 33 alunos. Cada aluno tem uma música e um cantor
favorito. Certo dia, cada um deles perguntou aos demais suas músicas e catores favoritos. Em seguida, cada
um falou dois números, o primeiro era a quantidades de alunos que gostavam da mesma música e o segundo,
a quantidade de alunos que tinham o mesmo cantor favorito. Sabe-se que cada um dos números de 0 a
10 apareceu entre as respostas. Mostre que existem dois alunos que gostam do mesmo cantor e da mesma
música.
Problema 124. (IMO 1983) Cada ponto do perı́metro de um triângulo eqüilátero é pintado de uma de duas
cores. Mostre que é possı́vel escolher três pontos da mesma cor formando um triângulo retângulo.
Problema 125. (Leningrado) Considere 70 inteiros positivos distintos menores ou iguais a 200. Prove que
existem dois deles cuja diferença é 4, 5 ou 9
Problema 126. Prove que de qualquer subconjunto de n + 1 elementos do conjunto {1, 2, ..., 2n} é possı́vel
escolher dois que sejam primos entre si.
Problema 127. Prove que de qualquer subconjunto de n + 1 elementos do conjunto {1, 2, ..., 2n} é possı́vel
escolher dois em que um seja divisı́vel pelo outro.
Problema 128. (USAMO 1985) Em uma festa há n pessoas. Prove que existem duas pessoas tais que, das
n − 2 pessoas restantes é possı́vel achar ⌊n/2⌋ − 1 onde cada uma delas conhece ou não conhecem ambas.
30 Prof. Bruno Holanda

Problema 129. (IMO 1972) Prove que, de qualquer conjunto de dez números distintos de dois dı́gitos,
podemos escolher dois subconjuntos A e B (disjuntos) cuja a soma dos elementos é a mesma em ambos.

Problema 130. (Jr. Balkan) Em um paı́s com seis cidades quaisquer duas são conectadas por uma linha
aérea (ida-volta). Cada linha aérea é operada por exatamente uma das duas empresas aéreas existentes.
Mostre que existem quatro cidades A, B, C, D tais que as linhas AB, BC, CD, DA são controladas por uma
única empresa.

Problema 131. (IMO 1947) Dezessete pessoas trocam cartas entre si, falando sobre exatamente um de três
assuntos. Prove que existe um grupo de três pessoas que falam sobre o mesmo assunto mutuamente.

Problema 132. Quarenta estudantes participaram de uma olimpı́ada de matemática. A prova consistia
de cinco problemas ao todo. Sabe-se que cada problema foi resolvido corretamente por pelo menos 23
participantes. Prove que deve existir dois participantes tais que todo problema foi resolvido por pelo menos
um deles dois.

Problema 133. Prove que em qualquer grupo de 17 números escolhidos do conjunto {1, 2, 3, ..., 24, 25} é
possı́vel escolher dois cujo produto é um quadrado perfeito.
-5-
Grafos

O que é um grafo? Se você nunca ouviu falar nisso antes, esta é certamente uma pergunta que você deve
estar se fazendo. Vamos tentar matar sua curiosidade contando como foi que a teoria dos grafos surgiu.

Figura 5.1: Mapa de Königsberg

A Literatura afirma que a teoria dos grafos começou na cidade de Königsberg em 1736 pelo grande
matemático suı́ço Leonhard Euler (1707-1783). A cidade era cortada pelo rio Pregel, que possuı́a duas ilhas
(figura 5.1). Como era muito complicado fazer o transporte de cargas e pessoas através de barcos, algumas
pontes foram construı́das para auxiliar neste deslocamento entre as ilhas e as duas margens. Após algum
tempo as pessoas começaram a se perguntar se era possı́vel sair de sua casa, passar por cada ponte exata-
mente uma vez e voltar para a segurança de seu lar.

Figura 5.2: Diagrama de Euler

31
32 Prof. Bruno Holanda

Para resolver o problema, Euler montou um diagrama que representasse o mapa da cidade. Ele o fez da
seguinte maneira: A cada ilha e margem ele associou a um ponto que chamaremos de vértice e a cada ponte
uma ligação que chamaremos de aresta. Com isso, ele obteve a figura 5.2:

Essa figura com vários pontos (vértices) e algumas ligações (arestas) que denominamos um grafo. Para
finalizar seu raciocı́nio, Euler percebeu que existiam vértices com exatamente três arestas incidentes. Por
outro lado, como os moradores queriam atravessar cada ponte apenas uma vez, cada vértice deveria ter
um número par arestas. Logo, se tornaria impossı́vel fazer um percurso seguindo as regras impostas pelos
moradores.

5.1 Conseitos Básicos

Como em toda teoria matemática, a teoria dos grafos está repleta de nomeclaturas e termos técnicos. Nesta
seção vamos aprender algumas definições importantes para o entendimento completo deste capı́tulo. A seguir
damos um exemplo de um grafo que representa um mapa de estradas e cidades.

F
D

A G
C

B E

Vamos aproveitar o grafo acima para abordar algumas definições. Por exemplo, o grafo acima é conexo,
pois é possı́vel ir de um vértice a qualquer outro passando usando algumas de suas arestas. Por exemplo,
para ir de A até G basta fazer a seguinte seqüência A → C → E → F → G. Dizemos então, que esta
seqüência é um caminho de A até G. Agora, um caminho fechado é chamado de ciclo. Por exemplo, o
caminho A → B → E → A é um ciclo de tamanho 3 (ou seja um C3 ). Já o ciclo B → E → G → F → B é
um C4 .

Outra notação muito importante é o grau. Vamos definir o grau de um vértice v como a quantidade de
arestas que incidem nele. E vamos denotar essa quantidade como d(v). Por exemplo, d(A) = 4, d(B) = 3 e
d(C) = 2. Os próximos exercı́cios servirão para fixarmos as definições que acabamos de aprender.

Exercı́cios:

1. Sabemos que o grafo anterior era conexo. Porém, existe uma aresta que, se retirada, o grafo passará a
ser desconexo. Que aresta é essa? Explique porque não pode ser outra.

2. Qual é o menor caminho de D até C? E o maior? (não se pode repetir arestas)

3. Quantos ciclos de tamanho três existem? E de tamanho quatro?


MÓDULO DE COMBINATÓRIA 33

4. Determine o ciclo que possui o maior tamanho.

5. Qual o vértice que tem o maior grau?

6. Calcule a soma dos graus de todos os vértices do grafo.

Você deve ter notado que o grafo de Euler possui uma particularidade: entre o mesmo par de vértices
existem duas arestas que os liga. Porém, a maioria dos grafos que estudamos são grafos simples. Ou seja,
grafos que não admitem laços (arestas que começam e terminam no mesmo vértice) e arestas múltiplas (como
no grafo de Euler).

O próximo problema é um dos mais famosos problemas de toda a olimpı́ada de matemática. Pode ter
certeza que você ainda vai ouvir falar desse problema muitas vezes.
Problema 134. É possı́vel que os cavalos da figura 1 fiquem na posição da figura 2?

Figura 1 Figura 2

Solução. Vamos enumerar as casas do tabuleiro da seguinte forma:

1 2 3

4 5 6

7 8 9

Agora vamos construir um grafo com vértices 1, 2, ..., 9 onde vamos ligar dois vértice i e j se é possı́vel o
cavalo ir da casa i até a casa j usando apenas um movimento. Dessa forma, obtemos o seguinte grafo:

1
6 8

7 5 3

2 4
9
34 Prof. Bruno Holanda

Agora colocamos os cavalos de acordo com os tabuleiros mostrados anteriormente.


1 1
6 8 6 8

7 5 3 7 5 3

2 4 2 4
9 9

Dessa forma fica fácil ver que é impossı́vel ir de uma configuração a outra, pois a ordem cı́clica dos cavalos
não pode mudar.

5.2 Grafos Simples

Teorema. Em um grafo simples G = (V, A), a soma dos graus de todos os seus vértices é igual ao dobro do
número de arestas. Ou seja; X
d(v) = 2 |A|
v∈V

Prova. De cada vértice v partem d(v) arestas. Porém, cada aresta possui dois vértices. Desse modo, se
somarmos os graus de todos os vértices obteremos o dobro do número de arestas.

Vamos aplicar esse teorema no problema que apareceu na olimpı́ada dos Estados Unidos de 1989.
Problema 135. Um torneio de xadrez reúne 20 jogadores. Foram jogadas 14 partidas, com cada jogador
jogando pelo menos uma vez. prove que nesse campeonato deve haver um conjunto de 6 jogos com 12 jo-
gadores diferentes.

Solução. Vamos montar um grafo G com 20 vértices a 14 arestas, onde os vértices representam os jogadores
e as arestas os jogos. Como cada jogador jogou pelo menos uma vez, cada vértice do grafo tem pelo menos
grau 1. Agora, usando o teorema temos que a soma dos graus dos vértices é 28. Daı́, pelo princı́pio da casa
dos pombos, devem existir pelo menos 12 vértices com grau exatamente 1. Esses 12 vértices representam os
12 jogadores solicitados pelo problema.

Veja que no exemplo anterior além de usar um teorema sobre grafos usamos também o princı́pio da casa
dos pombos. Usar outras idéias é muito comum em problemas de grafos. Pode aparecer de tudo, de contagem
ao método probabilı́stico. O próximo problema é da olimpı́ada do Leningrado de 1990. Neste exemplo vamos
usar uma idéia um pouco mais sofisticada, o princı́pio do extremo.
Problema 136. A Brunzundanga e a Zuzunzilandia são paı́ses vizinhos. Sabe-se que cada cidade está ligada
a no máximo dez outras cidades e que cidades do mesmo paı́s não são ligadas. Prove que podemos pintar
MÓDULO DE COMBINATÓRIA 35

essas estradas usando dez cores de modo que estradas adjacentes possuam cores distintas.
PS: As estradas são adjacentes se possuem uma cidade em comum.

Solução. Suponha que inicialmente todas as estradas estavam incolores. É claro que podemos escolher uma
delas e pintar com uma das cores. A partir daı́ vamos pintar as demais estradas respeitando a seguinte regra:

Sejam X e Y duas cidades (uma de cada paı́s) tais que a estrada XY está incolor. Desse modo, existe
uma cor (digamos a cor 1) que não foi usada em nenhuma das estradas partindo de X e uma cor (digamos a
cor 2) que não foi usada em nenhuma das estradas partindo de Y . Agora escolha o maior caminho da forma
2 − 1 − 2 − 1 − · · · partindo de X.

F F
1 2
2 1
=⇒
1 2

X 2 Y X 1 Y
2

Suponha, sem perda de generalidade, que esse caminho termine em uma aresta de cor 1 na cidade F .
Desse modo, não existe uma estrada de cor 2 partindo de F . Com isso, podemos trocar as cores das estradas
deste caminho (onde for 2 pintamos de 1 e virce-versa) sem nenhum problema. Para finalizar, basta pintar
a estrada XY da cor 2.

5.3 Árvores

Outro conceito muito importante em teoria dos grafos é a definição de árvore. Calma! Apesar da Natureza
ser um assunto muito importante, não vamos falar sobre o meio ambiente. Na verdade, uma árvore nada
mais é do que um grafo conexo sem ciclos. (Não tente falar sobre isto com seu professor de biologia!) Por
exemplo, os dois próximos grafos são árvores.

Note que nas duas árvores acima existem alguns vértices com grau exatamente 1. Esse vértices são con-
hecidos como as folhas da árvores. Além disso, um conjunto de árvores é chamado de floresta. Bem, mais
sugestivo que isso não poderia ser, não é?

Podemos citar três fatos sobre as árvores que merecem nossa atenção:
a. Toda árvore possui pelo menos duas folhas.
36 Prof. Bruno Holanda

b. Toda árvore com n vértices possui exatamente n − 1 arestas.


c. O “menor” grafo conexo é uma árvore.

Você deve estar se perguntando: Como assim menor grafo? Na verdade o fato fica mais claro se for
exposto da seguinte maneira: Se um grafo com n vértices é conexo, ele possui pelo menos n − 1 arestas.
Agora, para provar que isso gera uma árvore, é outra história.
. .
Vamos provar a utilidade das árvores (além de fabricar papel ⌣) resolvendo o seguinte problema que foi
o problema 6 do segundo nı́vel da olimpı́ada rioplatense de 2003:

Problema 137. Sobre uma mesa tem-se n ≥ 2 bolsas de plástico, todas de cores distintas. Cada uma está em
contato com a mesa ou está dentro de outra bolsa. A operação permitida é escolher uma bolsa que está em
contato com a mesa, retirar todas as bolsas do seu interior e coloca-las sobre a mesa e colocar todas as outras
bolsas que estavam fora e colocar no seu interior (sem modificar o conteúdo das outras bolsas). Determine o
total de configurações diferentes que podem ser obtidas utilizando a operação quantas vezes o necessário.

Solução. Construa um grafo com n vértices, onde cada vértice representa uma bolsa. Vamos ligar dois
vértices vi , vj se as bolsas bi , bj estão imediatamente uma dentro da outra. O grafo será algo semelhante ao
grafo abaixo.

Agora construa um novo vértice F e ligue-o a todos os vértices que representam as bolsas que estão sobre
a mesa. Note que aplicar a operação, no grafo representa trocar a posição do vértice F pela posição do vértice
que foi operado. E como o grafo possui um total de n + 1 vértices, existem ao todo n + 1 configurações.

=⇒
v4 v4
v1 v2 v3 F v2 v3

F v1

A figura acima mostra a troca das posições dos vértices F e v1 . Os vértices v1 , v2 , v3 representam as três
bolsas que estavam inicialmente sobre a mesa. Note que após aplicar a operação, os vértices que ficam ligados
a F são v1 e v4 , e as bolsas que ficam sobre a mesa são exatamente b1 e b4 .

Problema 138. Em um grupo de 50 cientistas sabe-se que cada um deles conhece pelo menos 25 outros
cientistas. Prove que podemos colocar quatro deles ao redor de uma mesa de forma que cada cientista esteja
sentado ao lado de dois amigos.
MÓDULO DE COMBINATÓRIA 37

Problema 139. Considere um grupo de 1997 pessoas. É possı́vel que cada uma delas conheça exatamente:
a) 3 pessoas?
b) 4 pessoas?

Problema 140. Considere um grupo de 1998 pessoas. É possı́vel que cada uma delas conheça exatamente
101 pessoas do grupo?

Problema 141. Cada um dos 102 estudantes é amigo de pelo menos 68 outros alunos. Prove que existem
quatro estudantes com o mesmo número de amigos.

Problema 142. Na Bruzundanga, quaisquer duas cidades são ligadas por uma estrada. Um imperador tirano
decidiu transformar todas essas estradas em estradas de mão única de tal forma que se uma pessoa sair de
sua cidade não poderá mais voltar. É possı́vel fazer tal crueldade?

Problema 143. Todos os vértices de um grafo têm grau 3. Prove que o grafo possui um ciclo.

Problema 144. A figura abaixo representa as ligações rodoviárias entre 14 cidades. Existe um caminho
passando por cada cidade exatamente uma vez?

Problema 145. Considere o quadrado 3 × 3 abaixo. Uma formiga sai do ponto A, caminha sobre as linhas
da grade e chega em B. Os únicos pontos por onde a formiga pode passar mais de uma vez são os os vértices
dos quadradinhos. Qual é o tamanho máximo do caminho que a formiga pode percorrer?
B

Problema 146. (Rússia 2003) Existem N cidades em um paı́s. Entre quaisquer duas cidades existe uma
estrada ou uma linha de trem. Um turista deseja viajar pelo paı́s, visitando cada cidade uma única vez, e
retornando à cidade inicial. Prove que ele pode escolher uma cidade, e percurso da viagem de tal forma que
ele não irá trocar de meio de transporte não mais do que uma vez.

Problema 147. Em uma festa havia 25 pessoas. Sabe-se que cada pessoa conhecia, na festa, exatamente k
pessoas. Por outro lado, quaisquer dois dos presentes na festa ou se conheciam, ou tinham pelo menos um
conhecido em comum. Determine o menor valor possı́vel de k.

Problema 148. (São Petersburgo 2001) Um paı́s possui 2000 cidades. Mostre que é possı́vel unir pares de
cidades usando estradas (duas-mãos) tal que para n = 1, 2, ..., 1000, existem exatamente duas cidades com
exatamente n estradas.
38 Prof. Bruno Holanda

Problema 149. (Leningrado 1990) Quaisquer duas das 101 cidade de Farland são conectadas por não mais
que uma estrada de mão única. Sabe-se que cada cidade possui 40 estradas “saindo” e 40 estradas “chegando”.
prove que uma pessoa pode sempre ir de uma cidade a outra passando por não mais do que outras duas
cidades.

Problema 150. Em um conjunto de n pessoas, em qualquer grupo de quatro delas existe uma que conhece
as outras três. Prove que existe uma pessoa que conhece todas as outras.

Problema 151. Em um conjunto de 2n pessoas existem duas com um número par de amigos em comum.

Problema 152. (Balcânica 1987) Em um congresso mundial, estão presentes 1985 pessoas. Em cada grupo
de três delas, existem duas que falam a mesma lı́ngua. Se cada pessoa fala no máximo cinco lı́nguas, mostre
que existe um grupo de 200 delas que falam a mesma lı́ngua.

Problema 153. (Irã 2003) Seja G um grafo simples com 100 arestas e 20 vértices. Podemos escolher um par
de arestas disjuntas de 4050 maneiras. Prove que o grafo é regular.

Problema 154. (IMO 1991) Suponha que G é um grafo conexo com k arestas. Prove que podemos enumerar
as arestas usando os números 1, 2, ..., k de modo que em cada vértice com mais de uma aresta, o m.d.c dos
inteiros escritos nas arestas que incidem nele seja 1.

Problema 155. (Leningrado 1988) Em Bilboland existem N cidades e 2N − 1 estradas, sempre de mão única,
ligando essas cidades; cada estrada lida apenas duas cidades. Em Bilboland podemos ir de uma cidade a
qualquer outra. Prove que existe uma estrada que pode ser interditada mas a propriedade acima continua
válida.

Problema 156. Raul vai dar uma festa com n pessoas (cada uma conhece pelo menos uma outra pessoa).
Victor chega e diz:
– Todas as pessoas que conhecem exatamente uma pessoa devem sair da festa.
Quando estas pessoas não estão mais na festa, Victor volta a falar:
– Todas as pessoas que conhecem exatamente duas pessoas devem sair da festa.
O procedimento se repete até n. Encontre o maior número de pessoas que pode restar no final do procedi-
mento.

Problema 157. Em um grafo com 100 vértices, em qualquer grupo de quatro vértices existe uma aresta que
liga dois deles. além disso, não há um caminho que passe por cada vértice exatamente uma vez. Prove que
podemos escolher dois vértices A e B de modo que cada um dos outros vértices está ligado ou a A ou a B.

Problema 158. (Torneio das Cidades 1982) Em certo paı́s existem mais do que 101 cidades. A capital deste
paı́s é conectada por linhas aéreas a outras 100 cidades, e cada cidade, exceto pela capital, é conectada a
outras 10 cidades (se A está conectado a B, B está conectado a A). Além disso, todas as linhas aéreas são
de uma única direção. Sabe-se que de qualquer cidade é possı́vel chegar a qualquer outra usando essas rotas.
Prove que é possı́vel fechar metade das linhas aéreas conectadas à capital, e preservar a capacidade de viajar
de uma cidade a qualquer outra.
MÓDULO DE COMBINATÓRIA 39

O Problema da Quatro Cores

O teorema das quatro cores, diz que todo mapa planar pode ser colorido usando quatro cores
de modo que paı́ses vizinhos possuam cores distintas é um problema antigo. Francis Guthrie
em 1852 propôs este problema ao seu irmão que era um estudante universitário de matemática
em Cambridge. Foi apenas em 1878 que o matemático Cayley o apresentou formalmente
na London Mathematical Society. Um ano depois, Kempe publicou uma prova incorreta
para o teorema que, em 1890, foi modificada por Heawood para provar o teorema da cinco cores.

Muitas foram as tentativas para solucionar o problema, muitas chegavam perto, mas a primeira
que se mostrou verdadeiramente correta foi a prova de Appel-Haken, de 1976. As idéias da
prova misturam idéias usadas por Kempe com a moderna capacidade de cálculo dos computa-
dores modernos. Primeiramente os grafos foram classificados em mais de 1400 configurações
que obedeciam a um mesmo critério e em seguida essas configurações foram testadas através
de um eficiente algoritmo de 741 páginas. Por conta dessa extensão em sua prova, alguns erros
foram encontrados, e mais configurações foram adicionadas à sua lista e em alguns anos de-
pois, a prova corrigida, K. Appel & W. Haken, Every Planar Map is Four Colorable, MAS 1989.

Uma recente prova de N. Robertson, D. Sanders, P.D. Seymour e R. Thomas, The four-colour
theorem, J. Combin. Theory B 70 (1997) é bem mais curta do que a primeira, mas usa
basicamente as mesmas idéias (além de um potente computador). Vinte cinco anos depois da
prova original, nenhuma prova sintética para o teorema das quatro cores foi encontrada.
-6-
Tabuleiros

Quem nunca brincou de quebra-cabeça? Temos várias “pecinhas” e temos que encontrar uma maneira de
unir todas elas para formar uma figura maior. O que costumava ser apenas um passa-tempo, ganhou uma
irmã que estudada por muitos matemáticos sérios pelo mundo a “Tiling Theory” (traduzindo: Teoria da
Cobertura). E por se tratar de um tema muito atrativo, logo ganhou força nas principais competições de
matemática.

6.1 Introdução

Problema 159. Determine se é possı́vel cobrir ou não o tabuleiro abaixo (sem sobreposições) usando apenas
dominós?

Solução. Pinte as casas do tabuleiro acima alternadamente de branco e preto (como no tabuleiro de xadrez).
Note que, não importa como colocamos o dominó no tabuleiro, ele sempre cobre uma casa branca e ou outra
preta. Desse modo se fosse possı́vel cobrir o tabuleiro usando apenas dominós, deverı́amos ter o tabuleiro
com a quantidade de casas pretas igual a quantidade de casas brancas. Mas no tabuleiro “quebrado” existem
18 casas brancas e 16 pretas. Logo, não é possı́vel fazer tal cobertura.

6.2 Os Poliminós

Do mesmo jeito que existem os dominós, também existem os polinimós.

Problema 160. Podemos cobrir um tabuleiro 10 × 10 usando apenas T-tetraminós como abaixo?

40
MÓDULO DE COMBINATÓRIA 41

Solução. Pinte o tabuleiro de branco e preto da maneira usual (como no xadrez). Note que ao colocarmos
um T-tetraminó no tabuleiro ele pode assumir colorações do tipo 1 ou 2.
Suponha que ao cobrir o tabuleiro usamos A peças do tipo 1 e B do tipo 2. Sabemos que devemos usar
25 peças no total ou seja A + B = 25. Cada peça do tipo 1 possui uma casa branca e cada peça do tipo 2
possui 3 casas brancas, e como temos ao todo 50 casas brancas no tabuleiro; A + 3B = 50. De modo análogo,
obtemos B + 3A = 50. Porém o sistema acima não possui solução inteira. Logo, não é possı́vel cobrir o
tabuleiro.

Tipo 1 Tipo 2
Se duas cores ajudam muita gente, quatro cores ajudam muito mais! É isso mesmo! Não vá pensando
que é só pintar o tabuleiro de preto e branco que você vai resolver todos os problemas de tabuleiro do mundo!
O próximo exemplo mostra que às vezes apenas duas cores não bastam.
Problema 161. É possı́vel que um cavalo do xadrez passe por todas as casas de um tabuleiro 4×10 exatamente
uma vez e, em seguida retorne para o quadrado original?

1 2 1 2 1 2
3 4 3 4 3 4
4 3 4 3 4 3
2 1 2 1 2 1

Solução. Pinte o tabuleiro 4 × n como mostra a figura acima. Assuma que seja possı́vel fazer que o cavalo
passe por todas as casas. Note que, se o cavalo está na casa 1 só poderá ir para casa 3 desse modo para o
cavalo ir para uma casa de cor 1 ele passa por duas casas de cor 3, e como cada cor possui o mesmo número
de casas, fica impossı́vel o cavalo fazer o passeio.

Vimos que pintar tabuleiros usando cores é uma exelente idéia. Uma idéia melhor ainda é pintar usando
números! Você deve estar se perguntando por que? Bem, os números possui propriedades aritméticas (i.e,
podem ser somados e multiplicados), coisa que não podemos fazer com cores. A não ser que você ache que
preto+branco=cinza.

Problema 162. (Estônia 1993) Para quais naturais n é posı́vel cobrir um retângulo de tamanho 3 × n com
peças mostradas na figura abaixo sem sobreposição?

Solução. Pinte o tabuleiro da seguinte forma:


Veja que a soma dos números cobertos por um L-trinimó é sempre 1 ou −1. Enquanto a soma dos
números cobertos por um Z-tetraminó é sempre zero. Agora para cubrir um tabuleiro 3 × n podemos usar
no máximo n peças,
Em geral, nos problemas de tabuleiro devemos seguir a seguinte estratégia:
42 Prof. Bruno Holanda

i) Tentar alguns casos particulares para ver se vai ou não ser possı́vel cobrir o tabuleiro.

ii) Se o tabuleiro for muito grande tente analisar casos pequenos (isso é útil em todo tipo de problema).

iii) Pintar. A pintura deve refletir as propriedades dos poliminós e do tabuleiro. Não se canse de tentar
novas pinturas e não se preocupe em usar apenas duas cores.

6.3 Problemas Propostos

Problema 163. Sobre uma das casas de um tabuleiro infinito, existe um cubo que cobre a casa perfeitamente.
A face no topo do cubo é branca, enquanto as demais faces são pretas. A cada passo, podemos tombar o
cubo para um dos lados. É possı́vel que:

a) Após 2004 passos o cubo volte ao mesmo quadrado com a face branca para baixo?

b) Após 2005 passos? (pp ??)

Problema 164. É possı́vel cobrir o tabuleiro a seguir usando apenas dominós?

Problema 165. É possı́vel cobrir um tabuleiro 8 × 10 usando apenas L-tetraminós como o abaixo?

Problema 166. É possı́vel cobrir um tabuleiro 5 × 10 usando apenas peças como abaixo? (pp ??)

Problema 167. Podemos cobrir uma caixa 10 × 10 × 10 com 250 caixas 1 × 1 × 4?

Problema 168. Um tabuleiro n × m foi totalmente coberto usando peças 4 × 1 e 2 × 2. Em seguida, todas
as peças foram retiradas do tabuleiro e uma peça 2 × 2 foi substituı́da por uma peça 4 × 1. Prove que o
tabuleiro não poderá ser mais coberto com essa troca.

Problema 169. De um tabuleiro n × n são retiradas suas quatro casas do quanto. Quais são os valores de n
para os quais esse tabuleiro quebrado é coberto por L-tetraminós?
MÓDULO DE COMBINATÓRIA 43

Problema 170. Sejam m e n inteiros maiores que 1. Se um tabuleiro m×n pode ser coberto com L-tetraminós
então é mn é múltiplo de 8. (pp ??)
Problema 171. (Teorema de Klarner) Um tabuleiro a × b pode ser coberto usando apenas peças 1 × n se e
somente se n | a ou n | b.
Problema 172. (Romênia 2000) Determine todos os tabuleiros m × n que podem ser cobertos usando L-
triminós como abaixo:

Problema 173. Um tabuleiro 7 × 7 é coberto usando 16 peças 3 × 1 e um monominó. Determine todas as


posições possı́veis do monominó.
Problema 174. (Estônia 2004) Um tabuleiro 5 × 5 é coberto por oito t-triminós e um monominó. Determine
todas as possı́veis possições que o monominó pode ocupar.
Problema 175. Retira-se uma casa de um tabuleiro 2n × 2n . Mostre que as 22n − 1 casas restantes podem
ser cobertas por L-triminós.
Problema 176. (Rioplatense 1999) É possı́vel cobrir um tabuleiro 1999 × 1999 com quadrados de lados
inteiros maiores que 35 e menores que 1999?
Ps: Os quadrados podem ser de tamanhos distintos.
Problema 177. Qual o número máximo de S-tetraminós como o abaixo podem ser colocados, sem so-
breposições em um tabuleiro 10 × 10?

Problema 178. Um tabuleiro 7 × 7 é coberto usando peças do seguinte tipo:

(1) (2) (3)


Prove que uma e apenas uma peça com quatro casas é usada.
Problema 179. (Rússia 1997) Podemos cobrir um tabuleiro 75 × 75 usando dominós e cruzes (como na figura
a seguir)? (pp ??)

cruz gancho
Problema 180. (Bielorussia 1999) Temos um tabuleiro 7 × 7 e peças dos três tipos a seguir:

(1) (2) (3)


44 Prof. Bruno Holanda

Samuel possui infinitas peças do tipo 2 e uma peça do tipo 3, enquanto Marcelo possui apenas uma peça
do tipo 1.

a) Prove que Marcelo pode colocar sua peça em algum lugar do tabuleiro de modo que Samuel não consiga
completar o resto do tabuleiro usando suas peças.

b) Suponha que Samuel adquiriu outra peça do tipo 3. Prove que não importa o lugar no qual Marcelo
coloque sua peça, Samuel sempre poderá completar o tabuleiro.

Problema 181. (IMO 2004) Um gancho é uma figura de seis casas como na figura acima ou qualquer uma
das figuras obtidas desta aplicando rotações ou reflexões. Determine todos os tabuleiros m × n que podem
ser cobertos usando esses ganchos.

Problema 182. (Putnam 1991) Existe algum natural L, tal que se m e n são inteiros maiores que L, então
todo tabuleiro m × n pode ser coberto usando peças 4 × 6 e 5 × 7?

Problema 183. (Bielorussia 2000) Ache o maior número de cruzes que podem cobir um tabuleiro 8 × 8.

Problema 184. (Bielorussia 2000) Ache o maior número T-hexaminós (como na figura abaixo) que podem
cobir um tabuleiro 9 × 9.

Problema 185. (Estônia 2004) Ache a medida do lado do menor cubo que pode ser coberto por crymbles
(figura 6.1). (pp ??)

Figura 6.1: Crymble

Problema 186. (Rússia 1996) Podemos cobrir um tabuleiro 5 × 7 com L-triminós que tal forma que cada
casa do tabuleiro seja coberta por um mesmo número de peças? (pp ??)

Problema 187. Determine se a última peça do resta um pode terminar na casa indicada (figura 6.2)
MÓDULO DE COMBINATÓRIA 45

Figura 6.2: Resta Um.

NOTAS:

i) São vários os paı́ses que costumam cobrar problemas de tabuleiro. Entre os muitos, destaco
a Estônia, a Bielorússia e a Rússia.

ii) Este capı́tulo foi baseado em um material feito pelo Carlos Shine e pelo Onofre Campos.
Hoje você pode encontrá-lo na internet.
-7-
Invariantes

Nesta aula vamos estudar o princı́pio da invariância. Ou seja, vamos resolver problemas que, dada uma
transformação, existe uma propriedade associada que nunca muda. Por exemplo, se somarmos dois a um
certo natural, sua paridade é invariante.

7.1 Analisando as invariantes

Quando você se deparar com um problema que usa algum tipo de transformação, a primeira
coisa que você deve tentar fazer é encotrar invariantes.

Problema 188. Cada um dos números a1 , a2 , ..., an é 1 ou −1, e temos que:

S = a1 a2 a3 a4 + a2 a3 a4 a5 + · · · + an a1 a2 a3 = 0

Prove que 4 | n.

Esse problema parece muito mais com um problema de teoria dos números do que um problema de
invariância. Na realidade, como isso pode ser um problema de invariância se, não temos nenhuma trans-
formação? Não seja por isso! Podemos criar nossas próprias transformações!

Solução. Nosso movimento será o seguinte: “trocar ai por −ai ”. Fazendo essa operação, a congruência de
S módulo 4 é invariante pois, trocam de sinal exatamente quatro parcelas de S. Assim, basta trocar todos
os ai ’s que forem −1 por 1. Portanto 0 ≡ S ≡ 1 + 1 + · · · + 1 ≡ n (mod 4) ⇒ 4 | n.

Outra maneira de se construir invariantes é através de energias. Os alunos que estudam fı́sica sabem
muito bem que “a energia total de um corpo é invariante em um sistema isolado”. Pois é, vamos usar esse
fato que lembra bastante invariantes em prol da matemática. Vamos construir nossas próprias energias e,
em seguida, mostrar que ela é invariante.

Problema 189. Em cada um dos dez degraus de uma escada existe uma rã. Cada rã pode, dando um pulo,
ir para outro degrau. Porém, quando uma rã faz isso, ao mesmo tempo, uma outra rã deve pular a mesma
quantidade de degraus em sentido contrário: uma sobe e outra desce. Conseguirão as rãs colocar-se todas

46
MÓDULO DE COMBINATÓRIA 47

juntas no mesmo degrau? Justifique.

Solução. Vamos dizer que uma rã tem energia i se ela estiver no i-ésimo degrau. Por exemplo, uma rã que
está no terceiro degrau tem energia 3, se ela pular para o sétimo degrau passará a ter energia 7. Note que a
soma das energias de todas as rãs é invariante, i.e, é sempre 1 + 2 + · · · 10 = 55. Desse modo se em algum
momento todas estiverem no mesmo degrau x, todas também terão energia x, ou seja 10x = 55. E como
x ∈ N, concluı́mos que é impossı́vel todas ficarem no mesmo degrau.

Com todos esses exemplos você deve ter percebido que existe um certo padrão nos problema de invariância.
Desse modo fica fácil identificá-los. Por outro lado, nem sempre é fácil achar a tal invariância. (blablabla)

Problema 190. (Russia 1995) Três pilhas de pedras estão sobre uma mesa. Sisyphus pode escolher duas
pilhas e transferir uma pedra de uma pilha para a outra. Para cada transferência ele recebe de Zeus o
número de moedas igual a diferença entre a quantidade de pedras da pilha de onde foi retirada a pedra e a
quantidade de pedras da pilha que receberá a pedra (a pedra na mão de Sisyphus não é levada em conta). Se
essa diferença for negativa, Sisyphus deve pagar a Zeus o número correspondente (o generoso Zeus permite
que ele pague depois se entrar em falência). Após algum tempo todas as pilhas voltaram a ter a mesma
quantidade inicial de pedras. Qual o número máximo de moedas que Sisyphus pode ter neste momento?

Solução.
Note que a maioria dos problemas de invariância têm o enunciado muito parecido. Todos eles de alguma
forma perguntam se, dado uma configuração é possı́vel chegar em outra. E como você também deve ter visto,
a maioria das respostas é sempre não. Cuidado! Existem problemas com o enunciado muito parecido mas, a
resposta é afirmativa. Nestes casos, devemos mostrar como chegar na tão desejada configuração.

O próximo problema é da olimpı́ada do Leningrado (região da Rússia que é atualmente conhecida como
São Petersburgo) de 1990. Esse exemplo irá esclarecer a idéia de “falsa invariante”.

Problema 191. O número 123 está na tela do computador de Teddy. A cada minuto o número escrito na
tela é somado com 102. Teddy pode trocar a ordem dos dı́gitos do número escrito na tela quando ele quiser.
Ele pode fazer com que o número escrito na tela seja sempre um número de três dı́gitos?
Solução. É possı́vel, basta ele seguir a seqüência: 123 → 225 → 327 → 429 → 531 ⇒ 135 → 237 ⇒ 327 →
429 · · · , onde → denota a operação de computador e ⇒ uma operação feita por Teddy.

Problema 192. Sete moedas estão sobre uma mesa mostrando a cara. Podemos escolher quaisquer quatro
delas e virá-las ao mesmo tempo. Podemos obter todas as moedas mostrando a coroa?

Problema 193. Os números 1, 2, 3, ..., 1989 são escritos em um quadro negro. Podemos apagar dois números
e escrever sua diferença no local. Após muitas operações ficamos apenas com um número. Esse número pode
ser o zero?

Problema 194. Os números 1, 2, ..., 20 são escritos em um quadro negro. Podemos apagar dois deles a e b e
escrever no lugar o número a + b + ab. Após muitas operações ficamos apenas com um número. Qual deve
ser esse número?
48 Prof. Bruno Holanda

Problema 195. Começando com a tripla {3, 4, 12} podemos a cada passo escolher dois número a e b e trocá-los
por 0.6a − 0.8b e 0.8a + 0.6b. Usando essa operação podemos obter {4, 6, 12}
Problema 196. Em um tabuleiro 8 × 8 uma das casas está pintada de preto e as outras casas de branco.
Podemos escolher qualquer linha ou coluna e trocar a cor de todas as suas casas. Usando essas operações,
podemos obter um tabuleiro inteiramente preto?
Problema 197. Em um tabuleiro 3 × 3 uma das casas do canto está pintada de preto e as outras casas de
branco. Podemos escolher qualquer linha ou coluna e trocar a cor de todas as suas casas. Usando essas
operações, podemos obter um tabuleiro inteiramente preto?
Problema 198. Em um tabuleiro 8 × 8 as quatro casas do canto estão pintadas de preto e as outras casas
de branco. Podemos escolher qualquer linha ou coluna e trocar a cor de todas as suas casas. Usando essas
operações, podemos obter um tabuleiro inteiramente preto?
Problema 199. Dado um polinômio quadrático ax2 + bx + c pode mos fazer as seguintes operações:
a. Trocar a com c.
b. Tocar x por x + t onde t é um real.
Usando essas operações é possı́vel transformar x2 − x − 2 em x2 − x − 1?
Problema 200. (Bulgária 2004) Considere todas as “palavras” formadas por a’s e b’s. Nestas palavras
podemos fazer as seguintes operações: Trocar um bloco aba por um bloco b, trocar um bloco bba por um
bloco a. Podemos fazer também as operações ao contrário. É possı́vel obter a seqüência b aa...a
| {z } a partir de
2003
aa...a
| {z } b?
2003

Problema 201. (Fortaleza 2003) Sobre uma circunferência tomamos m + n pontos, que a divide em m + n
pequenos arcos. Nós pintamos m pontos de branco e os n restantes de preto. Em seguida, associamos
a cada um dos m + n arcos um dos números 2, 1/2 ou 1, dependendo se as extremidades do arco sejam,
respectivamente, ambas brancas, ambas pretas ou uma preta e uma branca.
Calcule o produto dos números associados a cada um dos m + n arcos.
Problema 202. (Cone Sul 2000) No plano cartesiano, considere os pontos de coordenadas inteiras. Uma
operação consiste em escolher um destes pontos e realizar uma rotação de 90◦ no sentido anti-horário, com
centro neste ponto. É possı́vel, através de uma seqüência dessas operações, levar o triângulo de vértices
(0,0);(1,0);(0,1) no triângulo de vértices (0,0);(1,0);(1,1)?
Problema 203. (Leningrado 1988) Uma pilha com 1001 pedras está sobre uma mesa. Um jogo consiste
em escolher uma pilha sobre a mesa contendo mais de uma pedra, retirar uma pedra, e separar a pilha em
duas pilhas não vazias (não necessariamente iguais). Após vários movimentos, é possı́vel que todas as pilhas
restantes contenham exatamente três pedras?

7.2 Restos como invariantes

Na seção anterior vimos que algumas transformações mantêm algumas quantidades invariantes. Agora, va-
mos conhecer alguns problemas onde não mais as quantidades são invariantes e sim seus restos módulo m.
MÓDULO DE COMBINATÓRIA 49

Problema 204. (Leningrado 1987) As moedas dos paı́ses Dillia e Dallia são o diller e o daller, respectiva-
mente. Podemos trocas um diller por dez dallers e um daller por dez dillers. Zequinha possui um diller e
deseja obter a mesma quantidade de dillers e dallers usando essas operações. É possı́vel que isso ocorra?

Solução. Seja S a diferença entre a quantidade de dillers e dallers. Note que a congruência de S módulo
11 é invariante. Como inicialmente S ≡ 1 (mod 11), não se pode obter a mesma quantia de dillers e dallers.

Problema 205. (Rússia 1998) Um número de quatro dı́gitos é escrito no quadro-negro. As operações permi-
tidas são: adicionar 1 a dois dı́gitos vizinhos (caso nenhum deles seja 9), ou subtrair 1 de dois dı́gitos vizinhos
(caso nenhum deles seja 0). É possı́vel obtermos 2002 a partir de 1234 realizando algumas operações?

Problema 206. Seja d(x) a soma dos dı́gitos de x ∈ N. Determine todas as soluções de d(d(n)) + d(n) + n =
1997

Problema 207. (Torneio das Cidades ) Todo membro de uma seqüência, iniciando do segundo, é igual a
soma do termo anterior com a soma de seus dı́gitos. O primeiro número é 1. É possı́vel que 123456 pertença
à seqüência?

Problema 208. (Hong Kong 1997) Cinco números 1, 2, 3, 4, 5 estão escritos em um quadro negro. Um
estudante pode apagar dois dos números a e b e escrever nos seus lugares a + b e ab. Após algumas operações
podemos obter a quı́ntupla 21, 27, 64, 180, 540?

Problema 209. (Torneio das Cidades 1985) Na ilha de Camelot vivem 13 camaleões roxos, 15 verdes e 17
amarelos. Quando dois de cores distintas se encontram, mudam simultaneamente para a terceira cor. Poderia
dar-se a situação na qual todos tenham a mesma cor?

Problema 210. Em uma fábrica de cartões existem três máquinas. A primeira recebe um cartão (a, b)
e retorna um cartão (a + 1, b + 1). A segunda recebe um cartão (2a, 2b) e retorna um cartão (a, b). A
terceira recebe dois cartões (a, b) e (b, c) e retorna o cartão (a, c). Todas as máquinas também retornam o(s)
cartão(ões) dados. É possı́vel fabricar um cartão (1, 1988) se temos inicialmente apenas um cartão (5, 19)?

Problema 211. Com a calculadora KPK-1991 podemos efetuar duas operações: (a) elevar um número ao
quadrado; e (b) e obter de um número X de n dı́gitos (n > 3) o número A + B, onde A é o número formado
pelos três últimos de X e B o número formado pelos (n − 3) dı́gitos de X. Podemos obter o número 703 a
partir de 604 usando essa calculadora?

Problema 212. (Russia 1998) Temos um tabuleiro n × n (n > 100) com n − 1 casas iguais a 1 e o restante
iguais a 0. Podemos escolher uma casa, subtrair 1 dela, e adicionar 1 nas demais casas que estão na mesma
liha e coluna desta. Com essa operação, podemos fazer com que todas as casas do tabuleiro se tornem iguais?

7.3 Semi-invariantes

A idéia de semi-invariante é um pequena generalização da idéia de invariante. Diremos que uma propriedade
é semi-invariante quando ela muda de forma previsı́vel (periodicamente, sempre crescendo ou decrescendo).
Um exemplo bastante comum de semi-invariante é a idade de uma pessoa, que sempre cresce de forma
periódica (a cada 365).
50 Prof. Bruno Holanda

Problema 213. Nove casas 1 × 1 de um tabuleiro 10 × 10 estão infectadas. A cada segundo, uma casa que
possui duas casas vizinhas (com um lado em comum) infectadas também se torna infectada. É possı́vel todas
as casas se tornarem infectadas?

Solução. Veja que uma casa pode ser infectada de várias formas. Primeiramente vamos analisar a seguinte
“infecção”:

Figura 7.1: Infecção do Tipo 1.

Olhando para figura fica fácil observar que o perı́metro total da área infectada não muda após a infecção
do tipo 1. Desse modo, poderı́amos pensar que esse perı́metro é invariante e igual a 4 × 9 = 36. Daı́, como
o perı́metro do tabuleiro todo é 4 × 10 = 40 seria impossı́vel tornar o tabuleiro totalmente infectado. Mas
neste caso, estarı́amos cometendo um erro gravı́ssimo: esquecer de analisar todos os casos. Vejamos o que
acontece nos demais casos:

Figura 7.2: Infecção do Tipo 2.

Note que neste tipo de infecção o perı́metro não permanece constante, e sim diminui em duas unidades!
A princı́pio isso pode parecer um problema, mas não é. Se o perı́metro não aumenta, nunca poderá chegar a
40 (já que inicialmente ele é no máximo 36). Porém, para ter certeza que essa hipótese é verdadeira, ainda
temos que analisar o último caso:
Aqui podemos notar que o perı́metro fica menor ainda, diminuindo em quatro unidades. Com isso,
podemos concluir o problema. Ou seja, já que o perı́metro inicial é no máximo 36 (caso em que não há duas
casas infectadas vizinhas) e ele nunca cresce, jamais poderemos infectar completamente o tabuleiro.

Problema 214. Um total de 2000 pessoas estão divididas entre os 115 quartos de uma mansão. A cada
minuto, uma pessoa anda para um quarto com número igual ou maior de pessoas do qual ela estava. Prove
MÓDULO DE COMBINATÓRIA 51

Figura 7.3: Infecção do Tipo 3.

que eventualmente todas as pessoas vão estar em um mesmo quarto.

Solução. Sejam a1 , a2 , ..., a115 a quantidade de pessoas nos quartos 1, 2, ..., 115 respectivamente em um dado
momento. Defina I = a21 + a22 + · · · + a2115 .
Digamos que uma pessoa sai de um quarto com n pessoas e vai para um quarto com m pessoas (m ≥ n). A
variação de I é dada por:

∆I = ((m + 1)2 + (n − 1)2 ) − (m2 + n2 ) = 2(m − n + 1) > 0

Assim, toda vez que uma pessoa muda de quarto o valor de I cresce. Porém, sabemos que o valor de I não
pode crescer indefinidamente pois, o número de pessoas é finito. Ou seja, em um dado momento I não poderá
mais crescer, isso só acontecerá quando nenhuma pessoa puder mudar de quarto. Logo, todas elas deverão
estar no mesmo quarto.

Problema 215. (Leningrado ) Existem n ≥ 2 números não-nulos escritos em um quadro. Podemos escolher
dois números a e b e trocá-los por a + b/2 e b − a/2. Prove que após feito um movimento não podemos obter
os números iniciais novamente.
Problema 216. (Ucrânia 2000) Existem inicialmente
√ n números 1 escritos em um quadro. Em cada passo
ab 2
podemos apagar a e b e escrever o número no seu lugar. Após repetir essa operação n − 1 vezes, prove
a+b
1
que o último número escrito não pode ser menor que √
n
Problema 217. (São Petersburgo 1998) Um total de 119 anões vivem em uma aldeia com 120 pequenas
casas. Uma casa é dita super-habitada se 15 anões ou mais vivem nela. Todo dia, os anões de uma casa
super-habitada têm uma briga e se mudam para outras casas da aldeia. Algum dia, necessariamente se
encerrará?
Problema 218. (Russia 1997) Temos uma fileira longa de copos e n pedras no copo central (copo 0). Os
seguintes movimentos são permitidos:

Movimento tipo A:


i−1 i i+1i+2 i−1 i i+1i+2
52 Prof. Bruno Holanda

Se há pelo menos uma pedra no copo i e pelo menos uma no copo i + 1 podemos fazer uma pedra que
está no copo i + 1 pular para o copo i − 1 eliminando uma pedra do copo i.

Movimento tipo B:


i−1 i i+1i+2 i−1 i i+1i+2
Se há pelo menos duas pedras no copo i podemos pular uma pedra para o copo i + 2 e outra para o copo i − 1.

Demonstre o seguinte fato: fazendo os movimentos tipo A ou B durante um tempo suficientemente longo
sempre chegamos a uma configuração a partir da qual não é possı́vel fazer nenhum desses dois tipos de
movimento. Além disso, essa configuração final não depende da escolha de movimentos durante o processo.
Dica: Lembre-se de usar energia!
-8-
Princı́pio do Extremo

A idéia chave na solução de muitos problemas de combinatória, ou até mesmo em teoria dos números e álgebra
é a simples consideração de um elemento extremo (máximo ou mı́nimo). O próximo problema mostrará como
essa idéia pode ser simples e ao mesmo tempo poderosa.

Problema 219. (Leningrado 1988) Alguns pinos estão em um tabuleiro de xadrez. A cada segundo, um dos
pinos move para uma casa vizinha (lado em comum). Após muito tempo verificou-se que cada pino havia
passado todos todas as casas do tabuleiro exatamente uma vez e tinha voltado para a sua casa inicial. Prove
que existiu um momento em que todos os pinos estavam fora de sua casa inicial.

Solução. Seja P o primeiro pino que voltou para a sua posição inicial. Um movimento antes dele voltar
para sua casa, cada um dos outros pinos deve ter feito um movimento. De fato, se isso não fosse verdade, P
não poderia ter passado por todas as casas do tabuleiro. Desse modo, este será o momento em que todos os
pinos estarão em casas diferentes das iniciais.

Problema 220. (Teorema de Slyverstre) Um conjunto finito S de pontos no plano possui a propriedade que
qualquer reta que passa por dois destes pontos também passa por um terceiro. Prove que todos os pontos
estão sobre uma reta.

P0

l0 Q N M

Solução. Seja L o conjunto de todas as retas que passam por pelo menos dois pontos de S. Agora sejam
P0 ∈ S e l0 ∈ L tais que a distância entre P0 e l0 é a menor possı́vel porém, diferente de zero. Seja Q a
projeção de P0 sobre l0 . Como a reta l0 passa por três deles, pelo menos dois deles N e M estão na mesma
semi-reta (em relação a Q). Suponha que N é o mais próximo de Q desse modo, a distância entre N e a reta
P0 M é menor que a mı́nima. Contradição.

53
54 Prof. Bruno Holanda

Problema 221. Dado um conjunto de n pontos no plano, nem todos numa mesma reta, existe uma reta que
passa por exatamente dois desses pontos.
Problema 222. São dados n ≥ 3 pontos no plano de forma que quaisquer três estão em um triângulo de área
menor que 1. Mostre que todos eles estão em um triângulo de área menor que 4.
Problema 223. São dados n pontos no plano. Marcamos então, os pontos médios de todos os segmentos com
extremidades nesses n pontos. Prove que há pelo menos 2n − 3 pontos marcados distintos.
Problema 224. Há 20 paises em um planeta. Sabe-se que dentre quaisquer três desses paı́ses, existe sempre
dois sem relações diplomáticas. Prove que existem, no máximo, 200 embaixadas neste platena.
Problema 225. Todo participante de um torneio joga com cada um dos outros participantes exatamente
uma vez. Após o torneio cada jogador faz uma lista com os nomes de todos os jogadores vencidos por ele e
de todos os que foram vencidos pelos jogadores que ele venceu. Sabendo que neste torneio não há empates,
prove que existe um jogador cuja a lista possui o nome de todos os outros jogadores.
Problema 226. Em um pátio estão localizadas 2n + 1 pessoas tais que as distância entre quaisquer duas delas
são todas distintas. Em um dado momento cada uma delas atira na pessoa mais próxima de si. Prove que:
(a) Pelo menos uma pessoa irá sobreviver.

(b) Ninguém levará mais de cinco tiros.

(c) Os caminhos das balas não se encotram.

(d) Os segmentos formados pelas tragetórias das balas não formam um polı́gono convexo fechado.
Problema 227. Considere três escolas, cada uma com n alunos. Cada estudante tem ao todo n + 1 amigos
nas outras duas escolas em que ele não estuda. Prove que é possı́vel selecionar um estudante de cada escola
de tal forma que os três se conheçam mutuamente.
Problema 228. Em cada lattice point do plano é colocado um inteiro positivo. Cada um desses números é a
média aritmética de seus quatro vizinhos. Mostre que todos os números são iguais.
Problema 229. Cada casa de um tabuleiro 8 × 8 existe um número que pode ser 0 ou 1. Para cada casa que
contém um 0, a soma dos números escritos nas casas que estão ou na mesma linha ou na mesma coluna desta
casa é maior que ou igual a 8. Prove que a soma de todos os números no tabuleiro é maior que ou igual a 32.
Problema 230. O parlamento da Bruzundanga consiste de uma casa. Todo membro tem no máximo três
inimigos dentre os restantes. Mostre que é possı́vel separar a casa em duas casas de tal forma que cada
membro tenha no máximo um inimigo em sua casa.
Problema 231. (Leningrado 1989) Dado um número natural k maior que 1, prove que é impossı́vel colocar
os números 1, 2, ..., k2 em um tabuleiro k × k de forma que todas as somas dos números escritos em cada
linha e coluna sejam potências de 2.
Problema 232. (Torneio das Cidades 1983) Os números de 1 a 1000 são escritos ao redor de um cı́rculo.
Prove que é possı́vel formar 500 segmentos que não se cruzam, cada um ligando dois destes números, e de
tal modo que a diferença (em valor absoluto) entre dois números ligados não seja maior que 749.
55

Problema 233. (Torneio das Cidades 1985) Oito times de futibol participaram de um torneio com apenas
uma rodada onde cada time jogou contra todos os outros exatamente uma vez). Não houve empates. Prove
que após o termino do torneio é possı́vel escolher quatro times, digamos A, B, C, D tais que A derrotou B,
C e D; B derrotou C e D; e C derrotou D.

Você também pode gostar